100 Bai Tap Dai Cuong Huu Co [PDF]

  • 0 0 0
  • Gefällt Ihnen dieses papier und der download? Sie können Ihre eigene PDF-Datei in wenigen Minuten kostenlos online veröffentlichen! Anmelden
Datei wird geladen, bitte warten...
Zitiervorschau

Lời tựa Trong tủ sách của KEM hiện nay (www.tapchikem.com), bạn có thể dễ dàng tìm thấy hai bộ sách chính phục vụ cho việc tự học và bồi dưỡng HSG Quốc gia là Hành trình Olympiad (2200 trang, thay thế cho bộ “Tự học hóa”) và 55 Chuyên đề bồi dưỡng HSGQG (4400 trang). Cả hai bộ sách này tuy đầy đủ về mặt kiến thức, nhưng tương đối khó để tiếp cận – cả về nội dung lẫn giá thành – đặc biệt là với các độc giả mới chập chững tìm hiểu về chương trình Hóa học phổ thông chuyên. Đó là lí do series TOP100 ra đời: nhằm mang tới những ấn phẩm với GIÁ THÀNH RẺ và nội dung DỄ TIẾP CẬN hơn cho các độc giả của KEM. Quyển sách trên tay bạn là ấn phẩm đầu tiên về chủ đề Hóa học Hữu cơ, với nội dung chính được chia thành 4 phần: -

Phần 1: Cấu trúc – Hoạt tính. Phần 2: Hydrocarbon Phần 3: Dẫn xuất halogen, oxygen và nitrogen. Phần 4: Hợp chất carbonyl và dẫn xuất.

Với các phần 2-4, bài tập được biên soạn theo hướng ứng dụng các kiến thức từ phần 1, kết hợp với một số yếu tố mới lạ để giúp học sinh củng cố vững chắc hơn nền tảng kiến thức. Đối tượng sử dụng phù hợp là những học sinh đã tìm hiểu qua chương trình Hóa hữu cơ chuyên hoặc sinh viên ngành Hóa. Hi vọng rằng đây sẽ là một tài liệu hữu ích cho quý độc giả. Trân trọng! Ban biên tập Tạp chí KEM

1

Danh mục bài tập Cấu trúc – Hoạt tính ....................................................................... 5 1. Lai hóa – Độ hơn cấp ............................................................................................ 5 2. Cấu tạo và danh pháp ........................................................................................... 7 3. Danh pháp IUPAC ................................................................................................. 9 4. Danh pháp IUPAC ............................................................................................... 11 5. Đồng phân hình học ............................................................................................ 13 6. Đồng phân quang học ......................................................................................... 14 7. Đồng phân quang học ......................................................................................... 15 8. Hóa lập thể ......................................................................................................... 19 9. Cấu trúc và cấu dạng .......................................................................................... 23 10. Cấu dạng .......................................................................................................... 26 11. Cấu dạng .......................................................................................................... 28 12. Cấu dạng .......................................................................................................... 29 13. Những vấn đề về cấu dạng ................................................................................ 30 14. Sự quay trục ..................................................................................................... 31 15. Phân tích FMO .................................................................................................. 32 16. Tiểu phân trung gian ......................................................................................... 33 17. Cộng hưởng ...................................................................................................... 35 18. Cộng hưởng ...................................................................................................... 38 19. Tính thơm ......................................................................................................... 44 20. Tính thơm ......................................................................................................... 46 21. Tính thơm ......................................................................................................... 48 22. Ảnh hưởng của tính thơm .................................................................................. 50 23. COT và [COT]2- .................................................................................................. 52 24. Ảnh hưởng của tính thơm .................................................................................. 53 25. Cấu tạo và hoạt tính .......................................................................................... 54 26. Cộng hưởng ...................................................................................................... 56 27. Nhiệt độ nóng chảy ........................................................................................... 59 28. Tính tan ............................................................................................................ 61 29. Tính tan ............................................................................................................ 63 30. Ứng dụng của tính tan ....................................................................................... 66 31. Tính acid .......................................................................................................... 70 32. Tính acid .......................................................................................................... 71 33. So sánh tính base ............................................................................................. 78 34. Tính thơm và tính acid ....................................................................................... 82 35. Tính acid và base .............................................................................................. 83 36. Sự hỗ biến ........................................................................................................ 86 37. Cấu tạo và hoạt tính .......................................................................................... 88 38. Hóa lập thể của phản ứng .................................................................................. 91

Hydrocarbon ................................................................................92 1. Phản ứng thế gốc................................................................................................ 92 2. Điều chế alkene .................................................................................................. 95 3. Hydrogen hóa alkene .......................................................................................... 96 4. Phản ứng cộng electrophile ................................................................................. 98 5. Phản ứng cộng electrophile ............................................................................... 101

2

6. Quy tắc Markovnikov ......................................................................................... 105 7. Cộng electrophile .............................................................................................. 109 8. Cộng electrophile .............................................................................................. 110 9. Đề xuất cơ chế phản ứng ................................................................................... 111 10. Chuyển vị carbocation ..................................................................................... 113 11. Chuyển vị carbocation ...................................................................................... 115 12. Tổng hợp linalool ............................................................................................ 117 13. Chuyển vị carbocation ..................................................................................... 119 14. Hydrobor hóa alkene ....................................................................................... 120 15. Oxi hóa alkene ................................................................................................ 122 16. Phản ứng ozone phân và oxi hóa phân cắt ........................................................ 125 17. Phản ứng của epoxi ........................................................................................ 129 18. Chuyển hóa hữu cơ ......................................................................................... 130 19. Xác định hydrocarbon ..................................................................................... 131 20. Xác định hydrocarbon ..................................................................................... 132 21. Xác định hydrocarbon ..................................................................................... 133 22. Sơ đồ chuyển hóa ........................................................................................... 134 23. Alkyne ............................................................................................................ 137 24. Alkyne ............................................................................................................ 141 25. Tổng hợp cyclopropane ................................................................................... 146 26. Hệ liên hợp ..................................................................................................... 147 27. Phản ứng Diels-Alder....................................................................................... 151 28. Phản ứng Diels-Alder....................................................................................... 154 29. Phản ứng vòng hóa Diels-Alder ........................................................................ 156 30. Hiện tượng đồng phân lập thể của các hợp chất vòng ....................................... 158 31. Ảnh hưởng của tính thơm ................................................................................ 161 32. Cơ chế phản ứng ............................................................................................. 164

Dẫn xuất halogen, oxygen và nitrogen ............................................ 165 1. Tổng quan về phản ứng thế nucleophile ............................................................. 165 2. Tính nucleophile ............................................................................................... 169 3. Ảnh hưởng của dung môi .................................................................................. 172 4. Ảnh hưởng của nhóm rời đi ............................................................................... 174 5. Hóa lập thể ....................................................................................................... 177 6. Sản phẩm phản ứng thế .................................................................................... 178 7. Sản phẩm phản ứng thế .................................................................................... 183 8. Phản ứng thế nucleophile .................................................................................. 185 9. So sánh tốc độ phản ứng................................................................................... 186 10. So sánh tốc độ phản ứng ................................................................................. 187 11. Cơ chế phản ứng ............................................................................................. 189 12. Cơ chế phản ứng ............................................................................................. 193 13. Hóa lập thể của phản ứng thế .......................................................................... 196 14. Xác định hợp chất hữu cơ ................................................................................ 199 15. Chuyển hóa hữu cơ ......................................................................................... 200 16. Chuyển hóa hữu cơ ......................................................................................... 202 17. Ảnh hưởng của nhóm kề .................................................................................. 204 18. Cơ chế phản ứng ............................................................................................. 208

3

19. Cơ chế phản ứng ............................................................................................. 210 19A. Cơ chế phản ứng........................................................................................... 212 20. Cấu dạng và phản ứng tách ............................................................................. 213 21. Tổng hợp thyroxine ......................................................................................... 216 22. Tổng hợp amine .............................................................................................. 221 23. Xác định amine ............................................................................................... 224 24. Xác định amine ............................................................................................... 225

Hợp chất carbonyl ....................................................................... 226 1. Phản ứng của các aldehyde và ketone................................................................ 226 2. Các hợp chất carbonyl ở dạng chất phản ứng và sản phẩm.................................. 230 3. Nhận biết hợp chất hữu cơ ................................................................................ 232 3. Tổng hợp hữu cơ............................................................................................... 234 3A. Điều chế nhóm chức ester ............................................................................... 235 4. Các enol và enolate trong hóa hữu cơ ................................................................ 237 4A. Phản ứng ngưng tụ ......................................................................................... 241 5. Chuỗi chuyển hóa ............................................................................................. 242 6. Xác định hợp chất tự nhiên ................................................................................ 245 7. Ngưng tụ benzoin ............................................................................................. 248 8. Tổng hợp hữu cơ............................................................................................... 251 9. Tổng hợp hữu cơ............................................................................................... 254 10. Cơ chế phản ứng ............................................................................................. 257 11. Cơ chế phản ứng ............................................................................................. 259 12. Cơ chế phản ứng ............................................................................................. 262 13. Cơ chế phản ứng ............................................................................................. 265 14. Cơ chế phản ứng ............................................................................................. 268

4

Chương 1

Cấu trúc – Hoạt tính 1. Lai hóa – Độ hơn cấp 1) Trong phân tử 4-vinyl-3-methylhepta-1,2-diene-5-yne có bao nhiêu nguyên tử carbon lai hóa sp?

2) Xác định trạng thái lai hóa của các nguyên tử khác-hydrogen trong các cấu trúc dưới đây.

3) Hãy xác định nhóm thế nào có độ hơn cấp cao hơn trong số các nhóm sau đây:

5

Hướng dẫn 1) Có 3 nguyên tử carbon lai hóa sp.

2)

Chú ý rằng với chất B thì nguyên tử O nên được xem là lai hóa sp2 hơn là sp3 bởi theo đó thì cặp electron chưa liên kết trong orbital p sẽ tham gia vào sự cộng hưởng với liên kết đôi hiệu quả nhất. 3)

6

2. Cấu tạo và danh pháp i) Gọi tên gọi hợp chất CH3CH2C (CH3 )2 CH(CH3 )2 theo danh pháp IUPAC: (A) 3,3,5-Trimethylhexane. (B) 2,2,5-Trimethylhexane. (C) 2,4,4-Trimethylhexane. (D) 1,1,3,3-Tetramethylpentane. (E) Không phải các phương án trên. ii) Cấu dạng nào sau đây là của 2-methylpentane?

iii) Có bốn đồng phân cấu tạo ứng với công thức phân tử C4H9Br. Tên gọi nào dưới đây là của một trong bốn đồng phân đó: i) 1-Bromo-2-methylpropane. ii) 3-Bromobutane. iii) 2-Bromo-2-methylbutane. iv) 2-Bromo-1-methylpropane. iv) Gọi tên gọi hợp chất (CH3 )2 CHCH(CH3 ) CH2CH = CH2 theo danh pháp IUPAC: (A) 4,5-Dimethylhex-1-ene. (B) 4,5,5-Trimethylpent-1-ene. (C) 2,2-Dimethylhex-5-ene. (D) 4-Methyl-4-isopropylbut-1-ene.

7

Hướng dẫn i) 2,4,4-trimethylhexane (C). ii) Cấu dạng C. iii) Có 4 đồng phân cấu tạo ứng với công thức phân tử C4H9Br:

Trong số các tên gọi đã cho ở trên, chỉ có 1-bromo-2-methylpropane (A) là đúng. Phương án 3-bromobutane (B) sai vì phải là 2-… mới đúng. Phương án 2-bromo-2-methylbutane (C) sai vì tên gọi này ứng với công thức phân tử C5H11Br. Còn phương án 2-bromo-1-methylpropane (D) cũng không chính xác vì chọn sai mạch dài nhất. iv) Phương án (A).

8

3. Danh pháp IUPAC Biểu diễn công thức cấu tạo của các hợp chất có tên gọi như sau: a) b) c) d) e) f) g) h) i) j) k) l) m) n) o) p) q) r) s) t) u) v) w)

3-(iodomethyl)hexane 3-fluoro-2,3-dimethylpentane 5-(tert-butyl)-4-ethyl-2,3,4,7-tetramethyloctane 2-(allyloxy)prop-1-ene 1-cyclobutylcyclohept-1-ene 3,3,5-trimethylcyclohexan-1-one 5,6-dimethylhept-2-yne 1,2-dichlorocyclobutane 1-ethynyl-2-methylcyclopentane 3,4-dimethylocta-1,5-diyne 4-ethylheptan-3-ol 1-ethoxy-3-ethylheptane 3,4-dimethylocta-1,5-diyne 4-ethylheptan-3-ol 1-ethoxy-3-ethylheptane 2-ethylhexane-1,4-diol 3-isopropylhexan-2-ol 2-ethoxy-1,1-dimethylcyclopentane 2,2,2-tribromoacetaldehyde 3-hydroxypentanal 3,3,5,5-tetramethylheptan-4-one (E)-3-chlorohexa-1,4-diene (Z)-5-bromo-3-chlorohex-4-en-1-yne

9

Hướng dẫn

10

4. Danh pháp IUPAC Gọi tên các hợp chất sau đây theo danh pháp IUPAC:

11

Hướng dẫn

12

5. Đồng phân hình học i) Viết tất cả các cấu trúc của alkene có công thức C5H10 (bao gồm cả các đồng phân lập thể). Gọi tên chúng theo danh pháp IUPAC. ii) Cấu trúc nào sau đây là của (3E)-3,7-dimethylocta-1,3,6-triene?

iii) Xác định cấu hình các liên kết đôi của diene dưới đây:

(A) (2E,4E) (B) (2Z,4Z) (C) (2Z,4E) (D) (2E,4Z) Hướng dẫn i)

ii) Cấu trúc D ứng với tên gọi (3E)-3,7-dimethylocta-1,3,6-triene. iii) Cấu hình (2Z, 4Z) (B).

13

6. Đồng phân quang học i) Liệu cis-4-tert-butylcyclohexan-1-ol hay trans-4-tert-butylcyclohexan-1-ol có tính thủ tính?

ii) Hợp chất nào sau đây không có đồng phân quang học?

Hướng dẫn i) Cả cis-4-tert-butylcyclohexan-1-ol lẫn trans-4-tert-butylcyclohexan-1-ol đều không phải hợp chất thủ tính bởi phân tử của chúng đều có mặt phẳng đối xứng. ii) Chất C không có đồng phân quang học.

14

7. Đồng phân quang học i) Vẽ công thức chiếu Fischer cho (S) và (R)-2-iodobutane. ii) Xác định cấu hình tâm thủ tính của các phân tử sau:

iii) Vẽ công thức chiếu Fischer cho (2R, 3S)-2-bromo-3-chlorobutane và (2S, 3R)-2-bromo-3-chlorobutane, sắp xếp mạch carbon theo một đường thẳng. Dựa vào các tính chất vật lí (nhiệt độ sôi, nhiệt độ nóng chảy), liệu thể tách hỗn hợp đẳng mol của (2R,3S)-2-bromo-3-chlorobutane và (2S,3R)-2-bromo-3-chlorobutane thành các hợp chất riêng biệt hay không? Nếu có thì nên sử dụng kĩ thuật nào? Nếu không, hãy giải thích nguyên nhân. iv) Với mỗi cặp hợp chất sau, hãy chỉ ra mối quan hệ giữa chúng là đối quang, xuyên lập thể phân (đồng phân dia), đồng phân cấu tạo hay cùng là một chất?

15

v) Với mỗi cặp hợp chất sau, hãy chỉ ra mối quan hệ giữa chúng là: đồng phân cấu tạo, xuyên lập thể phân, đối quang hay không phải các mối liên hệ kể trên?

vi) Vẽ các đồng phân R và S của hợp chất dưới đây:

vii) Vẽ tất cả các đồng phân lập thể có thể có của hợp chất sau. Chỉ ra các cặp đối quang và xuyên lập thể phân.

Hướng dẫn i)

ii)

16

iii)

Các chất này là đối quang của nhau, nên chúng có cùng các tính chất vật lí như nhiệt độ sôi hay nhiệt độ nóng chảy. Sự khác biệt chỉ liên quan đến sự quay mặt phẳng ánh sáng phân cực. Do đó, không thể tách riêng chúng nếu chỉ dựa vào các tính chất vật lí. iv) A và B là cùng một chất, C và D là các xuyên lập thể phân, E và F là các xuyên lập thể phân, G và H là các đối quang, I và J là các đối quang, K và L là các đối quang, M và N là đồng phân cấu tạo. v) a) Xuyên lập thể phân; b) Giống nhau; c) Đối quang; d) Xuyên lập thể phân. vi)

Cách vẽ khác, cũng được chấp nhận:

17

vii)

18

8. Hóa lập thể 1) Methylphenidate (tên thương mại: Ritalin) được dùng trong điều trị chứng rối loạn tăng động giảm chú ý (ADHD). Ritalin là hỗn hợp của các đồng phân R,R và S,S mặc dù chỉ có đồng phân R,R là có hoạt tính điều trị ADHD. (Đối quang tinh khiết R,R, gọi là dexmethylphenidate, hiện nay được bán với tên thương mại Focalin.) Vẽ cấu trúc các đồng phân R,R và S,S của methylphenidate. 2) Artemisinin và mefloquine được sử dụng rộng rãi trong điều trị sốt rét. Hãy chỉ ra các tâm bất đối trong hai chất này và xác định cấu hình tuyệt đối của chúng (*) theo R/S. Chỉ rõ trạng thái lai hóa của các nguyên tử N trong mefloquine và cho biết nguyên tử nào bị proton hóa trong phản ứng của mefloquine với HCl.

3) Saquinavir (tên thương mại Invirase) thuộc nhóm thuốc ức chế protease, được sử dụng để điều trị HIV (virus gây suy giảm miễn dịch ở người). Hãy chỉ ra các tâm bất đối trong Saquinavir và xác định cấu hình tuyệt đối của chúng (*) theo R/S.

19

4) Vẽ đối quang của những hợp chất sau:

20

Hướng dẫn 1)

2)

Phản ứng của mefloquine với HCl:

21

3)

4)

22

9. Cấu trúc và cấu dạng 1) Vẽ cấu dạng bền của các hợp chất sau đây:

2) Người ta tìm thấy được một số gốc tự do A và B có cấu trúc như hình dưới. Cho biết cấu trúc hình học của nguyên tử cacbon trung tâm trong hai gốc này và giải thích tại sao nó có dạng hình học đó.

3) Trong cả ba trường hợp dưới đây, cấu dạng bên phải đều ưu thế hơn cấu dạng bên trái:

3) Khi khảo sát hợp chất sau thì thấy rằng với nhóm OH nằm ở vị trí biên (equatorial) thể hiện tính acid cao hơn so với lúc nhóm OH nằm ở vị trí trục (axial). Hãy giải thích nguyên nhân.

4) Xét cân bằng cấu dạng của hai ketone dưới đây. Trong dung môi DMSO thì cấu dạng A là cấu dạng ưu thế (100%), còn nếu sử dụng dung môi là 23

isooctane thì cấu dạng B lại là cấu dạng chủ yếu (22% A và 78% B). Giải thích lý do.

Hướng dẫn 1)

2) Do không thể đạt được cấu trúc phẳng vì sự cứng nhắc của vòng nên cả hai gốc tự do A và B đều có dạng hình tháp với nguyên tử cacbon trung tâm lai hóa sp3 3) -

Ở cân bằng đầu tiên cấu dạng bên trái chịu sức căng 1,3-allyl bất lợi:

-

Ở cân bằng thứ hai liên kết hydrogen tạo thành sẽ làm ổn định cấu dạng bên phải:

-

Ở cân bằng thứ ba cấu dạng bên trái chịu tương tác bất lợi giữa các nhóm methyl và hydrogen:

24

4) Có thể giải thích theo hai hướng: -

Nguyên tử hydrogen của nhóm OH đã bị ràng buộc bởi liên kết hydrogen nên khả năng phân li giảm. Anion sinh ra chịu đựng tương tác bất lợi với cặp electron tự do trên nguyên tử O trong vòng làm giảm tính bền dẫn đến sự giảm tính acid của chất đầu.

5) Ở cấu dạng A do tương tác lưỡng cực – lưỡng cực định hướng ngược nhau nên A là cấu dạng chủ yếu trong dung môi có tính phân cực như DMSO. Còn cấu dạng B mặc dù có tương tác không gian lớn (nhóm Me và i-Pr ở vị trí axial) nhưng do có liên kết hydrogen trong dung môi không phân cực có thể bù trừ cho tương tác lưỡng cực và tương tác không gian bất lợi, kết quả là cấu dạng B ưu thế hơn trong dung môi không cực.

25

10. Cấu dạng 1) Hãy cho biết đồng phân nào bền hơn trong hai chất sau đây và giải thích ngắn gọn

2) Hai hợp chất sau có hàm lượng enol tương ứng là 9.5% (Me2CHCHO chỉ có 0.014%) và 95%. Hãy giải thích khả năng tồn tại ở dạng enol của các chất này

3) Cho biết cấu dạng bền nhất của chất sau đây:

26

Hướng dẫn 1) Cấu dạng của đồng phân bền hơn:

Trong trường hợp này sẽ hạn chế tương tác 1,3-allylic đến mức tối thiểu 2) Giảm tương tác không gian khi tăng góc liên kết

3) Do có khả năng tạo thành các liên kết hydro nội phân tử giữa các nhóm COOH nên cấu dạng bền nhất của triacid sẽ là:

27

11. Cấu dạng 1) Các haloethane (CH3CH2X, X = Cl, Br, I) có hàng rào năng lượng quay xấp xỉ nhau (13.4 - 15.5 kJ/mol), mặc dù kích thước của các halogen tăng theo chiều Cl < Br < I. Giải thích. 2) Khi hai vòng 6 cạnh có chung một liên kết C-C thì hệ bicylic này được gọi là decalin. Có hai khả năng sắp xếp: trans-decalin có 2 nguyên tử hydrogen ở phần ngưng tụ (C-C) nằm khác phía, và cis-decalin có 2 nguyên tử hydrogen ở phần ngưng tụ cùng phía. Giải thích tại sao đồng phân trans- bền hơn?

Hướng dẫn 1) Mặc dù nguyên tử I lớn hơn nhiều so với Cl nhưng liên kết C-I cũng dài hơn nhiều so với liên kết C-Cl. Kết quả là tương tác che khuất của các nguyên tử H và I cũng không khác nhiều so với tương tác H, Cl.

2) Đồng phân cis kém bền hơn bởi tương tác trục (diaxial) 1,3 (xem hình.)

28

12. Cấu dạng 1) Chất nào trong số 2 chất sau đây phản ứng nhanh hơn với NaBH4? Giải thích.

2) Hợp chất X dưới đây có khả năng tác dụng với acid để giải phóng hydrogen. Hãy giải thích điều này dựa trên cấu trúc của nó.

Hướng dẫn 1) Cấu trúc decalin phản ứng nhanh hơn do sự chuyển dạng lai hóa từ Csp2 thành Csp3 dẫn đến sự giải tỏa tương tác che khuất (xem hình)

2) Cấu dạng của X được cho dưới đây. Lúc này carbocation hình thành ở carbon trung tâm sẽ được bền hóa mạnh do tương tác với ba cặp electron của ba nguyên tử N kế cận khiến cho liên kết C - H trung tâm rất dễ phân ly nên có thể tương tác được với acid để sinh ra H2.

29

13. Những vấn đề về cấu dạng 1) Đối với 3-phenylpropanoic acid, năng lượng của cấu dạng mà Ph và COOH che khuất nhau cao hơn so với cấu dạng mà Ph và COOH lệch nhau (syn); cấu dạng mà Ph và H che khuất nhau; cấu dạng mà Ph đối diện với COOH (anti) lần lượt là 15.0; 3.5; 18.5 kJ/mol. Hãy vẽ giản đồ năng lượng cho sự chuyển đổi các cấu dạng nói trên. 2) Xếp theo thứ tự tăng dần sức căng vòng với ba hệ thống: cyclopropane, oxirane và aziridine

Hướng dẫn 1) Giản đồ năng lượng chuyển đổi cấu dạng.

2) Khi tiến hành thay nhóm CH2 trong cyclopropane bằng các nhóm O hay NH thì số tương tác che khuất H/H giảm do lúc này H được thay thế bằng một cặp electron. Như vậy sức căng vòng sẽ tăng theo thứ tự oxirane < aziridine < cyclopropane.

30

14. Sự quay trục 1) Xét quá trình quay quanh trục C-N sau đây trong các amide. Năng lượng quan sát được của quá trình này là 20 kcal/mol. Giải thích

2) Đối với N,N-dimethylformamide thì các tính toán xác định năng lượng trong phase khí cho kết quả là 19.4 kcal/mol, thấp hơn 1.5 kcal/mol nếu tiến hành xác định trong dung dịch. Liệu cách giải thích ở ý (1) có thể áp dụng cho trường hợp này hay không? Giải thích. Hướng dẫn Do ảnh hưởng của hiệu ứng liên hợp làm xuất hiện liên kết đôi C=N gia tăng rào năng lượng, sự quay lúc này trở nên khó khăn hơn

Đối với DMF do cấu trúc lưỡng cực trong dung dịch bị solvate hóa nhiều hơn nên khả năng quay sẽ bị ảnh hưởng nhiều hơn do còn phải có sự phá vỡ solvate hóa. Về sự thủy phân các amide này thì những hợp chất mà ở đó nguyên tử N bị buộc vào một hệ thống căng hay hệ thống cứng nhắc thì khả năng liên hơp N–C=O sẽ giảm nên các tác nhân (H+ hay OH-) dễ tấn công vào hơn trong giai đoạn quyết định tốc độ phản ứng.

31

15. Phân tích FMO Mercury(II) acetate là một tác nhân oxi hóa nhẹ nhàng có thể oxi hóa hai muối ammonium bậc bốn I và II để tạo thành cation iminium tương ứng Ia và IIa:

Một điều thú vị là tốc độ của phản ứng oxi hóa I nhanh gấp 13 lần so với tốc k độ của phản ứng oxi hóa II: I = 13. Giải thích kết quả này. kII Hướng dẫn

H và nhóm HgOAc nằm ở vị trí đối song, do đó sự tương tác giữa MO sigma liên kết của C-H và MO sigma* phản liên kết của N-HgOAc diễn ra thuận lợi.

H và nhóm HgOAc không đối song với nhau, do đó sự tương tác giữa MO sigma liên kết của C-H và MO sigma* phản liên kết của N-HgOAc diễn ra không thuận lợi. Chính vì vậy mà có sự chênh lệch lớn về tốc độ giữa 2 phản ứng trên.

32

16. Tiểu phân trung gian i) Gốc tự do nào sau đây bền nhất?

ii) Carbocation nào sau đây bền nhất?

iii) Carbanion nào sau đây bền nhất?

iv) Carbocation (ion carbenium) bậc ba thì thường bền hơn carbocation bậc một và bậc hai, bởi: (A) có ba điện tích dương. (B) có cấu hình dạng tháp. (C) có cấu hình dạng tam giác phẳng. (D) có ba nhóm đẩy electron. v) Sắp xếp các carbocation sau theo thứ tự giảm dần tính bền:

33

vi) Vẽ cấu trúc gốc bền nhất được tạo thành từ sự tách một nguyên tử hydrogen từ hợp chất dưới đây.

vii) Sắp xếp các hợp chất sau theo thứ tự tăng dần khả năng dễ phân li (đồng li) thành các gốc tự do: 2,2,3,3-tetramethylbutane, 2,2-dimethyl-3,3,3triphenylpropane, hexaphenylethane, biphenyl. Hướng dẫn i) Gốc tự do bền nhất là B. ii) Carbocation bền nhất là B. iii) Carbanion bền nhất là A. iv) Thường thì carbocation bậc ba sẽ bền hơn carbocation bậc một và bậc hai bởi nó có ba nhóm alkyl đẩy electron (D). v) Điện tích dương trên nguyên tử carbon có thể được bền hóa hiệu quả hơn nhờ hiệu ứng cộng hưởng hơn là hiệu ứng siêu liên hợp. Do đó, cation benzyl được bền hóa hơn tất cả các carbocation khác. Trong số các carbocation còn lại thì càng nhiều nhóm alkyl sẽ càng giúp bền hòa điện tích dương, dẫn đến tiểu phân càng bền.

vi) Gốc C dưới đây bền nhất:

vii) Độ bền của các gốc tự do giảm theo trình tự: Ph3C•  Me3C•  Ph• Do đó, khả năng phân li thành các gốc tự do giảm theo trình tự sau: Ph3C–CPh3  Ph3C–CMe3  Me3C–CMe3  Ph–Ph 34

17. Cộng hưởng 1) Vẽ cấu trúc cộng hưởng của mỗi chất sau đây:

2) Một trong các chất p-dinitrobenzene hoặc m-dinitrobenzene thường được sử dụng làm chất bẫy gốc (radical trap) trong các phản ứng chuyển electron. Hợp chất tạo thành anion gốc bền nhất sẽ là bẫy tốt hơn. Hãy xét các anion gốc được tạo thành khi các chất đầu này nhận thêm một electron và dự đoán hợp chất nào thường được sử dụng. 3) Theo bạn sự giải tỏa được thể hiện trong các cấu trúc cộng hưởng sau có quan trọng không? Giải thích. Nếu điện tích dương được thay bằng điện tích âm thì có gì khác không?

Hướng dẫn 1) a)

b)

35

c)

d)

e)

Dưới đây là một vai trong số nhiều cấu trúc có thể được vẽ ra:

f)

36

2) Bắt đầu bằng cách vẽ cấu trúc của các hợp chất liên hợp ban đầu rồi sau đó thêm 1 electron vào để tạo thành anion gốc. Chỉ có một vài dạng cộng hưởng khả dĩ được vẽ ra. Tuy nhiên có thể thấy rằng cả anion và gốc đều có thể được giải tỏa đồng thời trên các nhóm nitro của p-dinitrobenzene, dẫn đến nhiều dạng cộng hưởng khả dĩ hơn. Bởi trung gian tạo thành từ hợp chất para được giải tỏa hơn nên sẽ dễ chuyển electron đến p-dinitrobenzene hơn và nó sẽ là chất bẫy gốc tốt hơn.

3) Các dạng cộng hưởng với điện tích dương trên nitrogen hoặc oxygen không đóng góp đáng kể vào sự bền hóa điện tích dương, bởi nitrogen và oxygen không có “lớp vỏ bát tử” (8 electron) khi chúng mang điện tích dương. Đây là điều cực kì quan trọng, khi oxygen và nitrogen có lớp vỏ bát tử, chúng có thể đóng góp đáng kể vào dạng lai hóa cộng hưởng mà chúng có điện tích dương. Ví dụ, carbocation 1-methoxyethyl được bền hóa đáng kể bởi oxygen kế cận:

Nếu nitrogen và oxyen có điện tích âm, chúng sẽ có vỏ bát tử. Điện tích âm trên các nguyên tử có độ âm điện lớn thế này sẽ được bền hóa đáng kể. Do đó, sự giải tỏa như trên sẽ rất quan trọng trong trường hợp điện tích âm.

37

18. Cộng hưởng 1) Cấu trúc cộng hưởng nào mô tả trạng thái cơ bản của hợp chất sau tốt hơn?

2) Vẽ cấu trúc cộng hưởng của các phân tử/ion sau:

3) Vẽ các cấu trúc cộng hưởng có thể có của mỗi hợp chất sau:

4) Các liên kết carbon-carbon trong naphthalene không bằng nhau. Sử dụng cấu trúc cộng hưởng để giải thích tại sao liên kết (a) ngắn hơn liên kết (b).

5) Khi xử lí indene với NaNH2 sẽ tạo thành base liên hợp của nó. Vẽ các cấu trúc cộng hưởng có thể có của base liên hợp của indene và giải thích tại sao pKa của indene lại thấp hơn của hầu hết các hydrocarbon.

38

6) Trong 5-methyl-1,3-cyclopentadiene (A) và 7-methyl-1,3,5-cycloheptatriene (B), nguyên tử H được đánh dấu nào có tính acid mạnh nhất? Nguyên tử nào có tính acid yếu nhất? Giải thích tại sao.

7) a) Giải thích tại sao phản ứng proton hóa pyrrole diễn ra ở C2 để tạo thành A thay vì ở trên 1 nguyên tử N để tạo thành B? b) Giải thích tại sao A có tính acid hơn C, acid liên hợp của pyridine.

8) Giải thích tại sao hơn chất A bền hơn hợp chất B nhiều?

9) Giải thích tại sao triphenylene giống với benzene khi không xảy ra phản ứng cộng với Br2 nhưng phenanthrene thì phản ứng được với Br2 tạo thành sản phẩm cộng (như hình vẽ)? (Gợi ý: Vẽ cấu trúc cộng hưởng của cả hai chất này và sử dụng chúng để xác định sự giải tỏa của các liên kết π.)

39

10) Độ dài liên kết đơn C–C thường nằm trong khoảng từ 150 – 154 ppm, tuy nhiên hợp chất có cấu trúc như dưới đây có một liên kết đơn C–C đặc biệt dài. Giải thích tại sao nó lại dài đến vậy?

Hướng dẫn 1) Cấu trúc bên phải bởi các nguyên tử đều đạt bát tử. 2)

40

3)

Ở trường hợp cuối cùng thì cấu trúc thứ 2 có sức căng vô cùng lớn. 4) Naphthalene có thể có 3 cấu trúc cộng hưởng:

2 trong số các cấu trúc cộng hưởng có (a) là liên kết đôi, (b) là liên kết đơn. Do đó, liên kết (b) có nhiều đặc trưng của liên kết đơn hơn nên nó dài hơn. 5) Base liên hợp của indene có 10 π electron nên nó có tính thơm và cực kì bền. Do đó, tính acid của indene mạnh hơn hầu hết các hydrocarbon.

41

6)

Hb có tính acid mạnh nhất bởi base liên hợp của nó có tính thơm (6 π electron). Hc có tính acid yếu nhất bởi base liên hợp của nó phản thơm (8 π electron). 7) a) Sự proton hóa ở C2 tạo thành acid liên hợp A bởi điện tích dương có thể được giải tỏa do cộng hưởng. Trong trường hợp B, điện tích dương không được bền hóa cộng hưởng.

b) Việc tách 1 proton khỏi A (không có tính thơm) sẽ để lại electron cho N và tạo thành pyrrole, có 6 π electron, được giải tỏa trong vòng 5 cạnh (hợp chất có tính thơm). Do đó sự deproton hóa của A thuận lợi. Trong khi đó, cả C và base pyridine liên hợp của nó đều có tính thơm. Do đó sự tách proton ra không có nhiều tác động lớn đến tính thơm của hệ nên C có tính acid kém hơn A. Thực tế thì pKa của A = 0.4 còn C = 5.2. 8) Cấu trúc cộng hưởng thứ hai của A cho thấy nó có vòng liên hợp hoàn toàn với 6 π electron, vòng này có tính thơm và rất bền. Tương tự, với cấu trúc B thì vòng liên hợp nhưng chỉ có 4 π electron (phản thơm), chính vì vậy rất kém bền.

42

9) Các cấu trúc cộng hưởng của triphenylene:

Các cấu trúc cộng hưởng A-H đều có 3 liên kết đôi và 3 liên kết đơn trong 3 vòng 6 cạnh. Điều này có nghĩa là mỗi vòng đều có tính chất như một vòng benzene cô lập, sẽ chỉ xảy ra phản ứng thế thay vì phản ứng cộng do mật độ π electron được giải tỏa đều. Chỉ có duy nhất cấu trúc cộng hưởng I không có dạng này. Mỗi liên kết C-C của triphenylene có 4 (hoặc 5) cấu trúc cộng hưởng mà trong đó nó là liên kết đơn và 4 (hoặc 5) cấu trúc cộng hưởng mà trong đó nó là liên kết đôi. Các cấu trúc cộng hưởng của phenanthrene:

Tuy nhiên, với phenanthrene, 4 trong số 5 cấu trúc cộng hưởng có 1 liên kết đôi ở các nguyên tử carbon được đánh dấu (chỉ có C là không.) Điều này có nghĩa là 2 nguyên tử C này có nhiều đặc trưng liên kết đôi hơn các liên kết C-C khác trong phenanthrene, do đó chúng dễ tham gia vào phản ứng cộng thay vì phản ứng thế. 10) Tránh tương tác lập thể giữa hai nhân benzene, hỗ trợ tăng độ bền phân tử.

43

19. Tính thơm 1) Cho biết mỗi tiểu phân sau đây có tính thơm, không thơm hay phản thơm: a)

b)

g)

c)

d)

f)

h)

i)

k)

2) Cho biết mỗi tiểu phân sau đây có tính thơm, không thơm hay phản thơm: a)

b)

c)

d)

f)

g)

h)

i)

44

e)

3) Cho biết mỗi tiểu phân sau đây có tính thơm, không thơm hay phản thơm: a)

b)

c)

d)

e)

f)

g)

h)

h)

i)

Hướng dẫn 1) Thơm: c, d, e, h; Không thơm: f, g; Phản thơm: a, b, i. 2) Thơm: b, d, g, h; Không thơm: c, e, i; Phản thơm: a, f. 3) Thơm: a, c, d, e, g; Không thơm: b, h; Phản thơm: f, i.

45

20. Tính thơm 1) Hợp chất nào sau đây có tính thơm:

2) Hợp chất nào sau đây có tính phản thơm:

3) Hợp chất nào sau đây có tính thơm:

46

Hướng dẫn 1)

2)

3)

47

21. Tính thơm 1) Hợp chất nào sau đây có tính thơm:

2) Hợp chất nào sau đây có tính phản thơm:

48

Hướng dẫn 1)

2)

49

22. Ảnh hưởng của tính thơm 1) Các phân tử được liệt kê dưới đây có moment lưỡng cực cao bất thường. Hãy giải thích tại sao.

2) Kết quả của các phản ứng sau là các sản phẩm kiểu-muối, bền (trong khí quyển trở) được tạo thành. Hãy chỉ ra các sản phẩm này và giải thích tính bền của chúng.

3) Phản ứng của azulene A với 1 đương lượng sulfuric acid sẽ tạo thành + hợp chất kết tinh C10H9  HSO4 − [B]. Hãy dự đoán cấu trúc của B.

50

Hướng dẫn 1) Moment lưỡng cực cao bất thường của các phân tử này (so với các alkene và ketone thông thường) gây ra bởi các cấu trúc cộng hưởng có tính thơm, thuận lợi về mặt năng lượng, với các điện tích tách biệt:

2) Các tiểu phân tạo thành (A, B, C, D) đều là các cấu trúc có tính thơm, thuận lợi về mặt năng lượng (bền).

3)

51

23. COT và [COT]2Cycloocta-1 3 5 7-tetraene (COT) được tổng hợp lần đầu tiên vào năm 1911. Khi tương tác với potassium, COT nhận thêm 2 electron theo kiểu cộng 1,4 và trở thành dianion với 2 nguyên tử carbon mang điện tích âm, kí hiệu là [COT]2-. Phương pháp nhiễu xạ tia X cho thấy COT có cấu trúc không phẳng, khoảng cách giữa các nguyên tử cacbon cạnh nhau lần lượt là 1.33 và 1.46 Å.Trong khi đó, dianion [COT]2- có cấu trúc phẳng, khoảng cách giữa các nguyên tử cacbon cạnh nhau đều bằng 1.41 Å. 1) Hãy vẽ cấu trúc dạng ghế và cấu trúc dạng thuyền của COT. So sánh độ bền của hai cấu trúc này và giải thích. 2) 2.Vẽ công thức cấu tạo của [COT]2-. Vì sao nó có cấu trúc phẳng và có các cạnh dài như nhau? Hướng dẫn 1)

Dạng thuyền

Dạng ghế

Dạng thuyền bền hơn dạng ghế, vì ở dạng thuyền, tất cả các liên kết đôi đều có cấu tạo phẳng. Trong khi đó, ở dạng ghế, hai liên kết đôi giữa C3 với C4, C7 với C8 không đồng phẳng, tạo ra sức căng, kém bền. 2) Với cấu trúc phẳng, [COT]2- bền vì trở thành hệ thơm (đó là một hệ liên hợp vòng khép kín, số electron π thỏa mãn qui tắc Hückel 4n+2), các cạnh có độ dài như nhau.

52

24. Ảnh hưởng của tính thơm 1) Hợp chất A tạo thành từ phản ứng dưới đây có moment lưỡng cực cao bất thường (13.5 D). Xác định cấu trúc của A và lí giải cho giá trị moment lưỡng cực của nó.

2) Cyclopentadiene-2,4-one, so với cyclohepta-2,4,6-trien-1-one, là hợp chất rất kém bền và dễ bị dimer hóa ở nhiệt độ thường. Giải thích cho sự khác biệt giữa hai chất này và viết cấu trúc dimer của cyclopentadien-2,4-one. Hướng dẫn 1) Hợp chất A chứa các mảnh cationic và anionic có tính thơm.

2) Cyclohepta-2,4,6-trien-1-one có các cấu trúc cộng hưởng có tính thơm, trong khi đó của cyclopentadiene-2,4-one lại là phản thơm.

53

25. Cấu tạo và hoạt tính 1) Mặc dù penicillin G có 2 nhóm amide nhưng có 1 nhóm hoạt động mạnh hơn nhiều so với nhóm còn lại. Nhóm nào hoạt động mạnh hơn? Tại sao?

2) Xác định các tâm electrophilic và nucleophilic trong mỗi phân tử sau:

3) Chỉ bằng cách lập luận dựa vào mật độ electron, hãy xác định em liệu các phản ứng sau có xảy ra hay không:

54

Hướng dẫn 1) Nhóm amide trong vòng 4 cạnh có góc liên kết 90o, dẫn đến sức căng vòng lớn, do đó sẽ có hoạt tính hóa học cao hơn. 2)

-

Trong d thì tất cả các liên kết C=C đều có tính nucleophile. Trong f thì tất cả các cặp electron chưa liên kết trên O và Cl đều có tính nucleophile.

3)

55

26. Cộng hưởng Giải thích các hiện tượng sau: a) b) c) d) e) f) g) h)

Các amide (R2NCOR) có tính nucleophile trên O mạnh hơn trên N. Nhóm ester có tính electrophile của C kém hơn so với trong ketone. Các acyl chloride (RCOCl) có tính acid mạnh hơn ester. Hợp chất 1 có moment lưỡng cực lớn hơn đồng phân 2. Hợp chất 3 có tính acid mạnh hơn 4. Imidazole 5 có tính base mạnh hơn đáng kể so với pyridine 6. Fulvene 7 có tính electrophile ở carbon gắn ngoài vòng (exocyclic). Cyclohexadienone 8 dễ bị hỗ biến (tautomer hóa) hơn đa số các hợp chất carbonyl. (Chú ý: Sự hỗ biến của các hợp chất carbonyl hầu hết đều diễn ra rất nhanh, do đó câu hỏi này là về xu hướng nhiệt động học, không phải động học.) i) Cyclopentadienone 9 cực kì kém bền. j) Chênh lệch pKa giữa PhSH và EtSH nhỏ hơn nhiều so với giữa PhOH và EtOH. k) Furan 10 chỉ tấn công electrophile ở C2, không ở C3.

Hướng dẫn a) Cả N và O trong amide đều có các cặp electron chưa liên kết, có thể phản ứng với electrophile. Khi O phản ứng với một tác nhân electrophile E+, sẽ tạo thành sản phẩm có 2 cấu trúc cộng hưởng tốt. Còn khi N phản ứng chỉ tạo thành sản phẩm có 1 cấu trúc cộng hưởng. Phản ứng trên O:

Phản ứng trên N:

56

b) Các ester có năng lượng thấp hơn ketone do sự bền hóa cộng hưởng từ nguyên tử O. Khi cộng một tác nhân nucleophile vào ester hoặc ketone, một tiểu phân trung gian tứ diện được tạo thành mà đối với nó sự cộng hưởng gần như không quan trọng. Do đó, sản phẩm tứ diện từ ester có năng lượng gần như giống với sản phẩm từ ketone. Kết quả là sẽ cần nhiều năng lượng để cộng một nucleophile vào ester hơn là ketone. c) Lập luận như trong ý b cũng có thể áp dụng vào trường hợp tính acid của acyl chloride so với ester. Chú ý rằng Cl và O có độ âm điện gần như bằng nhau, do đó chênh lệch về tính acid giữa acyl chloride và ester không phải do hiệu ứng cảm ứng mà bởi hiệu ứng cộng hưởng. d) Hợp chất 1 có một cấu trúc cộng hưởng mà trong đó các điện tích được tách riêng. Thông thường, một cấu trúc có các phần mang điện tách riêng thì có đóng góp nhỏ, nhưng trong trường hợp này, hai vòng được tạo thành có tính thơm, do đó chúng quan trọng hơn bình thường.

e) Khác biệt giữa 3 và 4 là bởi chất đầu có vòng. Việc mất 1 H có tính acid từ γ C của 3 tạo thành một tiểu phân có cấu trúc cộng hưởng thơm. 4 không có tính chất tương tự.

f) Cả imidazole và pyridine đều là những hợp chất thơm. Cặp electron chưa liên kết của N gắn với H trong imidazole là cần có để duy trì tính thơm, do đó nguyên tử N còn lại, có cặp chưa liên kết trong orbital sp2, là tâm base. Sự proton hóa nguyên tử N này tạo thành hợp chất có hai cấu trúc cộng hưởng tốt. Còn sự proton hóa pyridine tạo thành một hợp chất thơm chỉ có duy nhất một cấu trúc cộng hưởng tốt.

g) Các liên kết π C=C của các hydrocarbon đơn giản thường có tính nucleophile, không có tính electrophile. Tuy nhiên, khi một nucleophile tấn công vào một nguyên tử C ngoài vòng (exocyclic) của một hợp chất thơm

57

như fulvene thì các electron từ liên kết π C=C đi đến nguyên tử C trong vòng (endocylic) và tạo thành hệ thơm.

không thơm

thơm

h) Dạng hỗ biến của 2,4-cyclohexadienone, một hợp chất không thơm là phenol, một hợp chất thơm. i) Nhóm carbonyl C=O có hợp phần cộng hưởng C+-O- quan trọng. Trong cyclopentadienone, hợp phần cộng hưởng này là cấu trúc phản thơm. (Chính vì vậy, trong các đề xuất tổng hợp đừng lựa chọn cyclohexadienone hoặc cyclopentadienone làm nguyên liệu đầu!) j) PhOH có tính acid mạnh hơn đáng kể so với EtOH (pKa = 10 so với 17) do sự bền hóa cộng hưởng của base liên hợp trong PhOH. S lớn hơn O, do đó xen phủ S(p)-C(p) trong PhS- nhỏ hơn nhiều so với xen phủ O(p)-C(p) trong PhO-. Sự xen phủ giảm xuống trong PhS- dẫn đến sự bền hóa cộng hưởng bị gảm xuống, do đó, vòng phenyl gây ra khác biệt về tính acid của RSH ít hơn so với ROH. k) Sự tấn công của electrophile E+ vào C2 tạo thành một carbocation có 3 cấu trúc cộng hưởng tốt. Còn sự tấn công vào C3 chỉ tạo thành 2 cấu trúc cộng hưởng tốt.

58

27. Nhiệt độ nóng chảy Cho bảng số liệu sau: Acid béo

Stearic acid

Palmitic acid

Oleic acid

Linoleic acid

tnc, oC

69.6

63.1

13.4

5.2

a) Biết công thức phân tử của oleic acid là C18H34O2 có chứa 1 liên kết đôi ở dạng cis ở C9-C10 (C carboxyl là C1), công thức phân tử của linoleic acid là C18H32O2 có chứa 2 liên kết đôi đều ở dạng cis ở C9-C10 và C12C13 (C carboxyl là C1). Hãy viết công thức cấu trúc của oleic acid và linoleic acid. b) Hãy nêu nguyên nhân dẫn tới sự giảm dần nhiệt độ nóng chảy của 4 acid đã cho. c) Hãy giải tích vì sao chất béo thực vật thường có nhiệt độ đông đặc thấp hơn chất béo động vật?

59

Hướng dẫn Ở thể rắn các phân tử stearic acid có cấu trúc thẳng, gọn gàng, dễ sắp xếp chặt khít nên lực hút giữa chúng mạnh hơn, trong khi đó các phân tử oleic acid có cấu trúc uốn gập (ở chỗ cấu hình cis), cồng kềnh, khó sắp xếp chặt khít nên lực hút giữa chúng yếu hơn. Hình dưới đây cho thấy cùng một thể tích sẽ chứa được nhiều phân tử có cấu trúc thẳng hơn là phân tử có cấu trúc uốn gập:

Thành phần chính của chất béo thực vật là oleic acid vốn có tonc thấp.

60

28. Tính tan Dự đoán tính tan trong nước của các phân tử hữu cơ sau: a) DDT (thuốc trừ sâu, không phân b) Mestranol (thành phần trong hủy được) thuốc tránh thai)

c) Aspartame (chất làm ngọt nhân d) Caffeine (chất kích thích trong tạo tổng hợp) café, trà và nhiều loại nước giải khát)

e) Succrose (đường cát)

f) Carotatoxin (độc tố thần kinh phân lập được từ củ cà rốt)

61

Hướng dẫn a) DDT không có nitrogen, oxygen nên rất khó tan trong nước. b) Caffeine có nhiều liên kết phân cực chứa nitrogen, oxygen và có khả năng tạo nhiều liên kết hydrogen với nước nên có thể tan được. c) Mestranol có 2 nhóm chức phân cực nhưng số carbon lớn (10 C) nên không tan trong nước. d) Sucrose tương tự như caffeine (tan). e) Aspartame tương tự như caffeine (tan). f) Carotatoxin tương tự như mestranol (không tan).

62

29. Tính tan 1) Dự đoán tính tan của các vitamin sau trong nước và trong dung môi hữu cơ: a) Vitamin E.

b) Vitamine B6 (pyridoxine)

2) Giải thích tại sao A kém tan trong nước hơn B, mặc dù cả hai hợp chất này có các nhóm chức giống nhau.

3) Avobenzone và dioxybenzone là hai loại kem chống nắng thương mại. Sử dụng những nguyên lí về tính tan, dự đoán loại kem chống nắng nào dễ bị rửa trôi khi người dùng đi bơi. Giải thích lựa chọn của bạn.

63

Hướng dẫn 1) Về cơ bản, tính tan được xác định bởi độ phân cực. Các phân tử phân cực tan được trong nước, trong khi đó các phân tử không phân cực tan được trong dung môi hữu cơ. a) Vitamin E:

-

Chỉ có 2 nhóm phân cực. Có nhiều (29) liên kết C-C và C-H không phân cực.

 Tan được trong dung môi hữu cơ, khó tan trong nước. b) Vitamin B6 (pyridoxine):

-

có nhiều liên kết phân cực, ít liên kết không phân cực.

 Dễ tan trong nước. (Ngoài ra, nó cũng có thể tan trong dung môi hữu cơ bởi bản chất hữu cơ của mình, nhưng có khả năng là tan trong nước nhiều hơn.) 2) Trong A, các nhóm OH và CHO đủ gần nhau để tạo thành liên kết hydrogen nội phân tử. Trong B, hai nhóm này xa nhau. Do hai nhóm chức phân cực tham gia tạo liên kết hydrogen nội phân tử nên A khó tạo liên kết hydrogen với nước hơn B. Điều này dẫn đến độ tan trong nước của A < B (có sẵn hai nhóm chức để tạo liên kết hydrogen với dung môi H2O)

64

3) So sánh các nhóm chức trong 2 thành phần của kem chống nắng. Dioxybenzone có khả năng bị rửa bởi nước nhất nó chứa 2 nhóm hydroxy và tan trong nước nhiều hơn. -

Avobenzo: 2 nhóm ketone, 1 nhóm ether. Dioxybenzo: 2 nhóm hydroxyl, 1 nhóm ketone, 1 nhóm ether – tan trong nước tốt hơn.

65

30. Ứng dụng của tính tan 1) Poly(ethylene glycol) (PEG) và poly(vinyl chloride) (PVC) là những ví dụ tiêu biểu về các polymer - những phân tử hữu cơ lớn được tạo thành từ nhiều đơn vị giống nhau nhỏ hơn, nối với nhau bởi liên kết cộng hóa trị. Các polymer có những tính chất rất khác nhau phụ thuộc (một phần) vào các nhóm chức của chúng. Hãy đánh giá tính tan trong nước của mỗi polymer và giải thích tại sao PEG được dùng trong dầu gội còn PCV được dùng để chế tạo các ống dẫn nước.

2) Tetrahydrocannabinol (THC) là thành phần hoạt tính trong cần sa, còn ethanol là thành phần của đồ uống có cồn. Giải thích tại sao khi khám sàng lọc thuốc có thể phát hiện ra sự hiện diện của THC nhưng không phát hiện ra ethanol sau vài tuần kể từ khi các chất này được vào cơ thể?

3) Cocaine là một loại thuốc gây nghiện, bị lạm dụng nhiều. Cocaine thường được điều chế ở dạng muối hydrochloride (cocaine hydrochloride) nhưng có thể chuyển thành ma túy “crack” (phân tử trung hòa) bằng cách xử lí với base. Chất nào trong số hai hợp chất này (xem hình) có nhiệt độ sôi cao hơn? Chất nào tan trong nước tốt hơn? Dựa vào độ tan tương đối hãy giải thích tại sao crack thường được sử dụng bằng cách hút (hít), còn cocaine hydrochloride thì được bơm trực tiếp vào máu?

66

4) Không như xà phòng - là hợp chất ion - một số chất tẩy rửa dạng lỏng là các phân tử trung tính. Giải thích cách mỗi phân tử sau đây hoạt động để tẩy rửa vết bẩn như xà phòng. a)

b) Hướng dẫn 1) Do có các nguyên tử oxygen, PEG có khả năng tạo liên kết hydrogen với nước, giúp cho PEG tan được trong nước và phù hợp cho các sản phẩm như dầu gội. PVC không có liên kết hydrogen với nước, do đó PVC không tan trong nước, mặc dù nó có nhiều liên kết phân cực. Do PCV không tan trong nước nên có thể dùng để vận chuyển và giữ nước (ví dụ làm đường ống).

2) Các phân tử tan trong nước thì dễ bị bài tiết ra khỏi cơ thể (qua nước tiểu), trong khi đó các phân tử kém phân cực thì dễ hòa tan trong các mô mỡ (dung môi hữu cơ) và được giữ lại trong thời gian dài hơn. Đây chính là lí do tại sao khám sàng lọc có thể phát hiện được THC (không tan trong nước) nhưng không phát hiện được ethanol sau vài tuần.

67

3) So sánh các lực hút liên phân tử giữa crack và cocaine hydrochloride. Các lực hút liên phân tử mạnh hơn sẽ làm tăng cả nhiệt độ sôi lẫn độ tan trong nước.

Các phân tử này giống nhau ngoại trừ liên kết ion trong cocaine hydrochloride. Lực hút ion là lực cực kì mạnh, và do đó muối cocaine hydrochloride có nhiệt độ sôi cao hơn và độ tan trong nước lớn hơn. Do muối tan nhiều trong nước nên có thể tiêm trực tiếp vào máu. Crack được dùng bằng cách hút (hít) do nó có thể tan trong các mô hữu cơ của mũi và phổi. 4) Một chất tẩy rửa cần phải có cả đầu phân cực lẫn đầu không phân cực. Đầu phân cực sẽ tương tác với nước, còn đầu không phân cực sẽ bao xung quanh mỡ và chất hữu cơ. a)

68

Đầu không phân cực tương tác với chất hữu cơ

Đầu phân cực tương tác với nước qua các liên kết hydrogen

b)

Đầu không phân cực tương tác với chất hữu cơ

Đầu phân cực tương tác với nước qua các liên kết hydrogen

69

31. Tính acid Sắp xếp (có giải thích) theo trình tự tăng dần tính acid của các hợp chất sau:

CH3CH2COOH ; CH3CH2CH(CH3 )OH ; CH3CHBrCOOH ; CH3CH2NH2 Hướng dẫn Một cách để so sánh tính acid là so sánh khả năng phân ly cho proton H+, khả năng này tùy thuộc vào liên kết H–X– và ảnh hưởng của các nhóm liên kết với –X– của chất. Nếu các nhóm liên kết và bản chất của X làm cho liên kết H–X– kém bền, dễ bị cắt đứt thì H trở nên linh động, khả năng phân li cho proton càng dễ (tính acid càng mạnh). Trình tự tăng dần tính acid của các hợp chất:

Giải thích: -

-

-

Tính acid của A yếu nhất vì: So sánh về độ âm điện của nguyên tố X trong liên kết H-X- ; độ âm điện của oxygen lớn hơn nitrogen nên liên kết -O-H phân cực mạnh hơn liên kết -N-H, hydrogen trong A kém linh động hơn hydrogen trong các hợp chất khác trong dãy nên A có tính acid yếu nhất. Tính acid của B, C, D: Chất C và D có hiệu ứng cảm ứng của nhóm carbonyl làm O-H phân cực mạnh, đồng thời hiệu ứng liên hợp p -  giúp giải tỏa điện tích âm của ion carboxylate. Chất B có gốc secbutyl đẩy electron làm giảm sự phân cực của liên kết O–H trong B nên hydrogen kém linh động, C và D có tính acid mạnh hơn B. Tính acid của D mạnh hơn C vì: D có nguyên tử bromine hút electron (-I) làm cho hydrogen của nhóm OH cũng linh động, nên có tính acid mạnh hơn C.

70

32. Tính acid Đánh giá lực acid của mỗi nhóm hợp chất sau và sắp xếp các chất trong mỗi nhóm theo trật tự giảm dần lực acid. a) b) c) d) e)

HCO2H, CH3CO2H, CICH2CO2H, FCH2CO2H Phenol, m- và p-chlorophenol, m- và p-cresol Benzoic acid, m- và p-nitrobenzoic acid, m- và p-methoxybenzoic acid. 1,4-Pentadiene and cyclopentadiene

f)

g) CHF3 và CHCI3 h)

Hướng dẫn a) Khi xét đến lực acid, yếu tố quan trọng nhất cần xét đến là độ bền của base liên hợp so với acid. Nhìn chung, bất kì yếu tố nào làm tăng độ bền của base liên hợp so với acid thì sẽ làm tăng tính acid. Các thuật ngữ hiệu ứng cảm ứng và hiệu ứng cộng hưởng (mesomeric effect) được sử dụng xuyên suốt trong các thảo luận về tính acid và tính base. Hiệu ứng đầu tiên tác động qua khung σ của phân tử, còn hiệu ứng sau thì qua hệ π. Lực acid tương đối được đo bởi các giá trị pKa. Lực acid của một phân tử là phép đo liệu nó dễ mất proton tới mức nào; phân tử có lực acid càng mạnh thì mất proton càng nhanh. Với một acid dạng HA, chúng ta có thể viết cân bằng sau:

71

HA

H+ + A −

Trong một hợp chất có lực acid rất mạnh, đa số các proton có tính acid sẽ được giải phóng và cân bằng chuyển dịch sang bên phải. Nồng độ của H+ và A- cao, còn nồng độ của HA thấp. Do đó, hằng số cân bằng Ka ( H+   A −  ) lớn. Ngược lại, nếu một hợp có lực acid yếu, chỉ có một vài Ka = HA  proton được tách ra và cân bằng chuyển dịch sang bên trái. Do đó, hăng số cân bằng nhỏ. Do khoảng hằng số cân bằng rất lớn nên Ka thường không được sử dụng để đo trực tiếp tính acid, thay vào đó các giá trị pKa thường được dẫn ra. pKa của một hợp chất là logarithm âm của giá trị Ka (pKa = -log10Ka). pKa nhỏ thì ứng với lực acid mạnh, pKa lớn thì lực acid yếu. Cần lưu ý là do sử dụng thang logarith, nên một thay đổi nhỏ cũng thể hiện biến đổi hằng số cân bằng lớn (bởi pKa thay đổi 1 đơn vị là hằng số cân bằng đã biến đổi 10 lần.) a) HCO2H, CH3CO2H, ClCH2CO2H, FCH2CO2H Xu hướng biến đổi tính acid trong dãy các hợp chất X-CO2H thể hiện hiệu ứng cảm ứng của nhóm thế X. Anion tạo thành bởi sự mất 1 proton được bền hóa bởi các nhóm hút electron như fluorine và chlorine nhưng sẽ bị làm kém bền bởi các nhóm nhường electron như methyl. Do đó, FCH2CO2H có lực acid mạnh nhất trong dãy (pKa = 2.58), sau đó là ClCH2CO2H (pKa = 2.86), HCO2H (pKa = 3.75) và CH3CO2H (pKa = 4.76). b) Phenol, m- và p-chlorophenol, m- và p-cresol Phenol (pKa = 10.0) có lực acid mạnh hơn các alcohol không thơm do anion tạo thành bởi sự mất 1 proton được bền hóa qua sự giải tỏa điện tích âm khắp vòng benzene. Hiện tượng này được minh họa bởi các cấu trúc cộng hưởng sau đây:

Lực acid của các phenol thế có thể được giải thích dựa vào ảnh hưởng của nhóm thế trên mỗi cấu trúc cộng hưởng. Anion được tạo thành bởi sự mất 1 proton từ chlorophenol được bền hóa bởi hiệu ứng cảm ứng âm (hút

72

electron) từ nguyên tử chlorine, nên các chlorophenol có lực acid mạnh hơn phenol. Tuy nhiên, tình huống này phức tạp hơn thế: bởi nguyên tử chlorine với cặp lone pair (cặp electron chưa liên kết) có hiệu ứng liên hợp dương (đẩy electron) với vòng benzene và làm giảm độ bền anion1. Tác động này trở nên rõ ràng với p-chlorophenol (pKa = 9.38) hơn m-chlorophenol (pKa = 9.02).

Trong cấu trúc cộng hưởng thứ 3, lực đẩy điện tử giữa lone pair trên chlorine và nguyên tử carbon mang điện tích âm rõ ràng không thuận lợi, và đóng góp của cấu trúc này rõ ràng làm năng lượng bền hóa cộng hưởng giảm đi. Trong m-chlorophenol, hiệu ứng liên hợp dương có tác động làm bền hóa nhỏ hơn do không có cấu trúc cộng hưởng tương đương nào với điện tích âm trên meta carbon.

Trong các crezol (methylphenol), hiệu ứng cảm ứng dương của nhóm methyl làm giảm độ bèn của anion, nên các cresol có lực acid tháp hơn phenol. Tương tự như hiệu ứng liên hợp trong chlorophenol, hiệu ứng cảm ứng dương này trở nên rõ ràng với p-crezol (pKa = 10.27) hơn m-cresol (pKa = 10.9). c) Benzoic acid, m- và p-nitrobenzoic acid, m- và p-methoxybenzoic acid. Trong benzoic acid (pKa = 4.20), anion được tạo thành bởi sự mất 1 proton được bền hóa bởi sự giải tỏa điện tích âm trên 2 nguyên tử oxygen tương đương - được thể hiện qua 2 cấu trúc cộng hưởng dưới đây. Trái ngược với phenol, điện tích âm không thể được bền hóa bằng cách giải tỏa vào vòng benzene. Do lực acid của benzoic acid lớn hơn phenol nhiều, nên có thể kết 1

Các tác động ngược nhau của hiệu ứng cảm ứng âm và sự liên hợp nhường electron trong các benzene thế halo- cũng được quan sát thấy trong các phản ứng thế electrophile nhân thơm của chúng.

73

luận rằng sự giải tỏa trong hợp phần carboxylic acid hiệu quả hơn sự giải tỏa cộng hưởng của phenoxide anion vào vòng thơm.

Nhóm nitro là nhóm hút electron, và đo đó cũng làm bền hóa anion nên các nitrobenzoic acid có lực acid mạnh hơn benzoic acid. p-Nitrobenzoic acid (pKa = 3.44) có lực acid mạnh hơn một chút so với m-nitrobenzoic (pKa = 3.45). Điều này không có gì bất ngờ bởi sự tồn tại của cấu trúc cộng hưởng thứ ba dưới đây, với điện tích dương liền kề nhóm carboxylate giàu electron - bền hơn bất kì cấu trúc cộng hưởng tương tự nào của đồng phân meta. Chú ý rằng với mỗi cấu trúc cộng hưởng dưới đây, đều tồn tại một cấu trúc cộng hưởng tương đương của nhóm carboxylate.

Nhóm methoxy, cũng như nhóm thế chloro, có hiệu ứng cảm ứng âm (hút electron) nhưng lại có hiệu ứng liên hợp dương (nhường electron). Trái ngược với nhóm thế chloro, hiệu ứng liên hợp dương của nguyên tử oxygen mạnh hơn hiệu ứng cảm ứng âm nhiều. (Mặc dù độ âm điện của 2 nguyên tử tương đương nhau, nhưng bán kính nguyên tử nhỏ hơn của oxygen giúp sự xen phủ orbital với nguyên tử carbon tốt hơn, do đó hiệu ứng liên hợp mạnh hơn.) Các methoxybenzoic acid có lực acid yếu hơn benzoic acid. Lực acid của p-methoxybenzoic acid (pKa = 4.47) yếu hơn một chút so với mmethoxybenzoic acid (pKa = 4.09) do cấu trúc cộng hưởng thứ ba (dưới đây),

74

có nguyên tử carbon mang điện tích âm cạnh nhóm carboxylate anion - kém bền hơn bất kì cấu trúc tương đương nào của đồng phân meta.

d) 1,4-Pentadiene và cyclopentadiene. 1,4-Pentadiene (pKa = 35) có lực acid mạnh hơn hydrocarbon no tương ứng, vì anion tạo thành bởi sự mất 1 proton được bền hóa cộng hưởng:

(Việc đo pKa của 1,4-pentadiene không dễ thực hiện, do anion này tồn tại trong cân bằng với cả 1,4-pentadiene lẫn 1,3-pentadiene [bền về mặt nhiệt động học hơn]. Do đó, giá trị pKa trên là tính toán được chứ không phải đo trực tiếp.) Phân tử phẳng cyclopentadiene (pKa = 15.0) thậm chí còn có lực acid mạnh hơn bởi sự mất 1 proton sẽ tạo thành hệ vòng 6 electron (thơm), được bền hóa cộng hưởng tương tự như benzene:

e) Trong mỗi hợp chất trên, proton có lực acid mạnh nhất đều thuộc về nguyên tử carbon đánh dấu hoa thị (*) và anion tương ứng được bền hóa bởi sự tạo thành enolate ion, có thể được biểu diễn bởi các cấu trúc cộng hưởng sau đây:

75

Cả 3 nhóm thế R đều khiến cho anion bền hơn so với dạng enolate không liên hợp, bởi điện tích âm có thể được giải tỏa tiếp. 2 nhóm keto làm cho anion có độ bền lớn nhất (pKa = 8.9) bởi sự giải tỏa trên 5 nguyên tử, gồm 2 nguyên tử oxygen âm điện.

Hợp chất cyano có lực acid nhỏ hơn (pKa = 10.0) do nitrogen có độ âm điện thấp hơn oxygen:

Nhóm thế ester làm bền hóa kém nhất (pKa = 11.0) do sự giải tỏa điện tích âm vào nhóm carbonyl (ester) lại được bị giảm chút bởi hiệu ứng liên hợp dương của lone pair từ oxygen:

f) Trong phân tử đầu tiên (pKa > 40), sự tạo thành enolate ion rất không thuận lợi: khi loại bỏ 1 proton, orbital có điện tích âm nằm vuông góc với các carbonyl π orbital. Dạng hình học này ngăn cản sự xen phủ orbital để tạo thành một hệ π mở rộng:

76

Cấu trúc cộng hưởng II (dưới đây) - vi phạm quy tắc Bredt - có đóng góp rất nhỏ bởi dạng hình học của phân tử không thuận lợi cho sự xen phủ carboncarbon π.

Với trường hợp thứ hai, enolate ion có thể được tạo thành và hợp chất này có lực acid mạnh hơn (pKa = 20):

g) CHF3 và CHCI3 Do độ âm điện của fluorine lớn hơn nhiều so với chlorne nên F3C- được dự đoán là bền hơn nhiều so với dẫn xuất chlorine tương ứng, và CHF3 có thể có lực acid mạnh hơn CHCl3 rát nhiều. Tuy nhiên, thực tế thì CHCl3 có lực acid lớn hơn, do Cl3C- được bền hóa bởi sự giải tỏa điện tích âm trong các d orbital của chlorine. Trái lại, nguyên tử fluorine không có d orbital nên F3C- kém bền hơn nhiều. Do đó, lực acid của CHF3 (pKa = 32) thấp hơn CHCl3 (pKa = 24) khá nhiều. h) Dù có độ âm điện thấp hơn nhưng hợp chất của sulfur (pKa = 35) lại có lực acid mạnh hơn chất tương ứng chứa oxygen (pKa > 40). Điều này được giải thích bởi khả năng mở rộng vỏ bát tử của sulfur (sử dụng các orbital d), tạo thành các cấu trúc cộng hưởng sau:

77

33. So sánh tính base So sánh tính base của các hợp chất nitrogen trong mỗi cặp chất sau (xét trong dung dịch nước). Giải thích câu trả lời dựa trên các hiệu ứng lập thể hoặc các yếu tố không gian. a)

b)

c)

d)

e)

f)

g)

h)

i)

78

Hướng dẫn a)

N-methylaniline là base yếu hơn benzylamine do hiệu ứng cộng hưởng +M của nhóm amino liên kết trực tiếp với vòng benzene. Trong benzylamine, hiệu ứng này không được thể hiện, do nhóm amino tách biệt với vòng thơm (cách bởi nhóm -CH-). Đồng thời, benzylamine là base yếu hơn methylamine (pKa 10.62, pKb 3.38) do hiệu ứng -I của nhóm C6H5. b)

Cyclohexylamine có tính base mạnh hơn sáu bậc (106) so với aniline do hiệu ứng cộng hưởng +M của nhóm amino trên vòng benzene. c)

Khác biệt giữa tính base giữa pyrrolidine và pyrrole là khoảng 15 bậc (1015). Thực tế thì pyrrolidine cũng là một amine bậc hai phổ biến, và trong pyrrole thì cặp electron chưa liên kết của nitrogen cũng là một phần thuộc hệ thơm (6 electron) của dị vòng, do đó không tham gia phản ứng proton hóa – phản ứng này được diễn ra ở vị trí thứ hai của vòng, ở nguyên tử carbon.

79

d)

p-toluidine có tính base mạnh hơn bốn bậc so với p-nitroaniline, do nhóm nitrogen làm gia tăng mức độ liên hợp của cặp electron trên nguyên tử nitrogen của nhóm nitro với vòng benzene, dẫn đến tính base giảm mạnh. e)

Quinuclidine là amine bậc ba được cố định bởi hệ bicyclic, có tính base mạnh. Trong pyridine thơm, cặp electron chưa liên kết của nguyên tử nitrogen thuộc orbital lai hóa sp2, nằm vuông góc với hệ thơm (6 electron pi). Do đó, pyridine có tính base thấp hơn 5 bậc so với quinuclidine, có cặp electron thuộc orbital lai hóa sp3. f)

Aniline có tính base mạnh hơn bởi cặp electron chưa liên kết của nó tham gia vào hệ liên hợp (hiệu ứng +M) với một vòng thơm, trong khi đó diphenylamine thì là với hai vòng thơm.

80

g)

Piperidine có tính base mạnh hơn pyridine khoảng 6 bậc, do cặp electron chưa liên kết của nitrogen thuộc orbital lai hóa sp3, trong khi đó của pyridine là sp2. h)

Tính base của p-hydroxyaniline và p-toluidine tương đối gần nhau (chênh lệch dưới 10 lần). n-hydroxyaniline (p-aminophenol) có tính base mạnh hơn một chút do hiệu ứng +M của nhóm OH làm giảm hiệu ứng +M của nhóm amino, dẫn đến tính base của nguyên tử nitrogen trong chất này mạnh hơn. i)

1,8-bis (dimethylamino naphthalene (còn gọi là "bọt xốp (hấp thụ) proton") là base khá mạnh, do ở dạng proton hóa thì hydrogen tạo thành liên kết hydrogen mạnh với nguyên tử nitrogen của nhóm amino liền kề.

81

34. Tính thơm và tính acid Giải thích sự khác nhau về tính acid C-H của các proton methylene trong các hợp chất sau:

pKa

22.9

18.5

15

36

Hướng dẫn Tính acid C-H của các proton methylen trong các hợp chất này liên quan đến độ bền của carbanion, tức là các base liên hợp, được tạo thành khi tương tác với base. Trong hợp chất B, tính acid của các proton methylene mạnh nhất, do sự deproton hóa (tách proton) dẫn đến sự tạo thành anion cyclopentadienyl thơm; còn trong hợp chất D, có tính acid yếu nhất, thì khi deproton hóa tạo thành anion tropylium phản thơm. Các hợp chất A và B, với tính acid nằm ở khoảng giữa, thì về mặt hình thức có mảnh cyclopentadienyl trong cấu trúc, nhưng sự bền hóa carbanion tạo thành sẽ phá vỡ hệ thơm bền của các vòng benzene.

82

35. Tính acid và base 1) a) Fumaric acid và maleic acid có các hằng số phân ly nấc 1 (k1), nấc 2 (k2). Hãy so sánh các cặp hằng số phân ly tương ứng của hai acid này và giải thích. b) Tại sao squaric acid lại có pK2 nhỏ của cả fumaric acid và maleic acid?

2) Giải thích tại sao: a) Hợp chất dưới đây có tính acid rất mạnh (thậm chí còn mạnh hơn cả nitric acid):

b) Tính base của chất B mạnh hơn A:

c) 1,1,3,3-tetramethylguanidine (pKa ≈ 12,00) là base mạnh hơn N,Ndimethylacetamide (pKa ≈ 0,50) rất nhiều.

83

3) Puberulic acid (tên gọi IUPAC: 4,5,6-trihydroxy-3-oxocyclohepta-1,4,6triene-1-carboxylic acid) là chất kháng sinh có trong một loài nấm, vừa có tính acid vừa có tính base. Hãy giải thích nguyên nhân xuất hiện tính acid và tính base của puberulic acid.

Hướng dẫn 1) a) k1(M) > k1(F) là do M có khả năng tạo được liên kết hydrogen nội phân tử, liên kết O-H của M trong qúa trình phân ly thứ nhất phân cực hơn so với F và base liên hợp M’ cũng bền hơn F’.

k2(M) < k2(F) do liên kết hydrogen nội phân tử làm cho M’ bền, khó nhường proton hơn so với F’. Ngoài ra base liên hợp M’’ lại kém bền hơn (do năng lượng tương tác giữa các nhóm –COO- lớn hơn) base liên hợp F’. b) pK2 của squaric acid nhỏ hơn pK2 của fumaric acid và maleic acid là do dianion của squaric acid tạo được hệ thống liên hợp bền vững:

2)

84

a)

Anion sinh ra được bền vững bằng hiệu ứng siêu liên hợp âm của nhóm CF3, nhưng nhân tố quan trọng hơn đóng góp vào tính bền của anion chính là sự tạo thành hệ thơm bền vững (6e). b)

Trong cấu trúc A có sự liên hợp giữa cặp electron trên N và nhóm C=O làm giảm tính base. Điều này không tồn tại trong B do với cấu trúc vòng cứng nhắc không thể tồn tại sự xen phủ hiệu lực giữa obitan chứa cặp e trên N với nhóm CO, vì thế tính base của B mạnh hơn. c) Do cation sinh ra từ guanidine được ổn định mạnh bằng cộng hưởng. 3) Chất này acid vì có nhóm -COOH và các nhóm- OH dạng enol nên có khả năng phân li tạo thành H+. Có tính base vì có khả năng nhận H+ tạo cacbocation bền là vòng có tính thơm (theo qui tắc Hückel).

85

36. Sự hỗ biến 1) Tại sao pKa của các proton Ha trong 1-acetylcyclohexene cao hơn pKa của các proton Hb?

2) Tại sao 5,5-dimethyl-1,3-cyclohexanedione chủ yếu tồn tại ở dạng enol nhưng 2,2-dimethyl-1,3-cyclohexanedione thì không?

86

Hướng dẫn 1) Sự tách loại Ha tạo thành 2 cấu trúc cộng hưởng mà điện tích âm không bao giờ ở trên O. Sự tách loại Hb tạo thành 3 cấu trúc cộng hưởng mà một trong số đó có điện tích âm ở trên O, làm cho base liên hợp bền hơn. Chính vì vậy Hb có tính acid mạnh hơn (pKa thấp hơn).

2) 5,5-dimethyl-1,3-cyclohexanedione chủ yếu tồn tại ở dạng enol bởi liên kết C=C của enol liên hợp với nhóm C=O khác. Sự liên hợp làm bền hóa enol này. Enol của 2,2-dimethyl-1,3-cyclohexanedione không liên hợp với nhóm carbonyl còn lại.

87

37. Cấu tạo và hoạt tính 1) So sánh độ dài liên kết C=O trong ba hợp chất sau đây và giải thích ngắn gọn:

2) Giải thích tốc độ dung môi phân của các hợp chất sau đây trong acetic acid: X

Tốc độ tương đối

H

1

OMe

130

Me

5

Cl

0.2

3) Giải thích hướng của phản ứng sau đây:

4) Khi hoàn tan đồng phân cis và trans của hợp chất dưới đây vào acid sunfuric 60% thì chỉ mỗi đồng phân cis tạo dung dịch màu vàng đậm, còn đồng phân trans cho dung dịch trong suốt. Hãy giải thích điều này.

88

Hướng dẫn 1) Độ dài liên kết các chất tăng dần theo thứ tự:

Để so sánh độ dài liên kết ta phải xét khả năng cộng hưởng của cặp e trên dị tố N vào C=O. Ở đây A không thể tham gia cộng hưởng (vi phạm quy tắc Bredt) nên liên kết C=O hoàn toàn là liên kết đôi nên A phải có liên kết C=O ngắn nhất. C dài hơn A nhưng ngắn hơn B do khả năng tham gia cộng hưởng của N tốt hơn O, thành ra liên kết C=O trong B có nhiều tình chất liên kết đơn hơn nên dài hơn. 2) Phản ứng dung môi phân có sự tạo thành carbocation không cổ điển (hình vẽ), nó được ổn định hóa bởi các nhóm dồn electron dẫn đến bảng kết quả như trên

3)

Phản ứng tạo thành carbocation không cổ điện được ổn định hóa bằng liên kết đôi qua tương tác không gian. Điều này dẫn đến việc mặt sau của liên kết C - OTs đã bị chắn nên phản ứng chỉ có thể xảy ra theo hướng bảo toàn cấu hình.

89

4) Ở đồng phân cis tạo thành carbocation được ổn định bằng obitan pi của liên kết đôi, cộng thêm việc nhóm phenyl trợ màu dẫn đến tạo thành dung dịch có màu. Đồng phân trans không có sự hỗ trợ từ liên kết đôi bên vòng cyclohexene nên không có màu:

90

38. Hóa lập thể của phản ứng i) Phản ứng chlor hóa (S)-2-methyl-1-chlorobutane, được kích phát bởi tia tử ngoại, dẫn đến sự tạo thành một hỗn hợp mà từ đó có thể phân lập được 2-methyl-1,2-dichlorobutane và 2-methyl-1,3-dichlorobutane.Dựa vào quan điểm của bạn, hãy đánh giá hóa lập thể của các sản phẩm tạo thành. ii) Dẫn ra các công thức chiếu Fischer cho các đồng phân 2,3dichlorobutane tạo thành khi chlor hóa racemic 2-chlorobutane dưới ánh sáng tử ngoại. Chỉ ra cấu hình R, S của mỗi nguyên tử carbon bất đối. Hướng dẫn i) Các gốc tự do không bảo toàn cấu hình tứ diện. Do đó, phản ứng monochlor hóa (S)-2-methyl 1-chlorobutane quang hoạt ở vị trí số 2 sẽ tạo thành 2-methyl-1,2-dichlorobutane dạng racemic. Khi chlor hóa vị trí số 3 thì cấu hình của carbon C2 được bảo toàn, dẫn đến sự tạo thành hỗn hợp xuyên lập thể phân của (2R, 3R)- và (2R, 3S)-2-methyl-1,3-dichlorobutane:

ii)

91

Chương 2

Hydrocarbon 1. Phản ứng thế gốc i) Xác định công thức sản phẩm chính của phản ứng bromo hóa ethylcyclohexane và đề xuất cơ chế tạo thành nó.

ii) Trong mỗi cặp chất sau, hãy chỉ ra hợp chất có nguyên tử hydrogen (được kí hiệu bởi các chữ cái) có hoạt tính mạnh hơn trong phản ứng halogen hóa gốc. a)

b)

iii) Phản ứng bromo hóa các alkene diễn ra chậm hơn nhiều so với chlor hóa. Xác định sản phẩm chính của phản ứng giữa 2-methylbutane với bromine khi chiếu sáng hoặc đun nóng.

iv) Xác định sản phẩm chính của phản ứng monobromo hóa 2,2dimethylbutane.

92

v) Sắp xếp các hợp chất trong dãy sau theo thứ tự tăng dần hoạt tính trong phản ứng bromo hóa cơ chế gốc.

vi) Hợp chất nào sau đây không được tạo thành trong phản ứng bromo hóa gốc tự do 1-ethyl-4-methylbenzene.

Hướng dẫn i) Phản ứng diễn ra qua gốc bậc ba bền hơn.

ii) а) Hа > Hb; b) Hc > Hd.

93

iii) Sản phẩm chính trong phản ứng của 2-methylbutane với bromine khi chiếu sáng hoặc đun nóng là bromide B. Do hoạt tính của bromine không quá mạnh nên phản ứng bromo hóa diễn ra có độ chọn lọc cao, alkyl bromide được tạo thành chủ yếu là bởi sự thế diễn ra ở liên kết C-H hoạt tính nhất (dễ phân li ra gốc bền nhất).

iv) Sản phẩm chính là hợp chất C, do độ chọn lọc của phản ứng bromo hóa cao và sự tác kích của bromine diễn ra theo hướng tạo thành gốc bền nhất.

v) D  A  B  C. Thứ tự hoạt tính có mối tương quan với độ bền trung gian gốc. Gốc tạo thành từ hợp chất D không được bền hóa bởi sự liên hợp với vòng thơm.

vi) Trong phản ứng bromo hóa gốc tự do của 1-ethyl-4-methylbenzene không thể tạo thành hợp chất D, là sản phẩm bromo hóa vào vòng benzene.

94

2. Điều chế alkene i) Phản ứng nào sau đây không phù hợp để điều chế but-1-ene.

ii) Đề xuất cách điều chlorocyclohexane.

chế

trans-2-chlorocyclohexanol

đi

từ

Hướng dẫn i) Phản ứng B không phù hợp bởi theo quy tắc Zaitsev thì sản phẩm chính tạo thành sẽ là but-2-ene. ii)

95

3. Hydrogen hóa alkene i) Hợp chất nào sau đây có nhiệt hydrogen hóa cao nhất?

ii) Trong dãy các bicycloalkene sau, hãy chọn ra hợp chất giải phóng nhiệt lượng lớn nhất trong phản ứng hydrogen hóa.

iii) Hãy chọn ra điều kiện tối ưu nhất để thực hiện chuyển hóa của 1(chloromethyl)cyclohex-1-ene thành (chloromethyl)cyclohexane.

(A) H2 /Pt − C /EtOH (B) 1. NaBH4 /EtOH, 2. H3O+ (C) 1. LiAlH4 /Et2O, 2. H3O+ . (D) 1. (CH3 )2 CHCH2  AlH, 2. H3O+ . 2 iv) Phản ứng hydrogen hóa xúc tác một hydrocarbon thủ tính C6H12 tạo thành sản phẩm là hydrocarbon phi thủ tính C6H14. Hydrocarbon C6H12 ban đầu là (A) Cis-hex-2-ene (B) 3-Methylpent-2-ene (C) 4-Methylpent-2-ene (D) 3-Methylpent-1-ene

96

Hướng dẫn i) Hydrogen hóa là phản ứng tỏa nhiệt. Alkene càng có nhiều nhóm alkyl liên kết với các nguyên tử carbon lai hóa sp2 thì càng bền, dẫn đến nhiệt hydrogen hóa càng thấp. Độ bền tương đối của các alkene:

Giá trị nhiệt hydrogen hóa thực tế (theo kcal/mol) của các hợp chất đã cho là: 26.6 ( C )  26.9 ( D )  27.6 ( A )  28.6 (E )  30.3 (B ). ii) Hợp chất A giải phóng nhiệt lượng lớn nhất khi hydrogen hóa hoàn toàn bởi alkene này kém bền nhất (giải thích tương tự ý i ở trên). iii) H2 / Pt − C / EtOH ( A ) iv) 3-Methylpent-1-ene (D)

97

4. Phản ứng cộng electrophile i) Hợp chất nào sau đây không phải là electrophile:

AlCl3 , C2H5OC2H5 , BF3 , (CH3 )3 C+ , HOCl. ii) Alkene nào sau đây có thể phản ứng nhanh nhất với các tác nhân electrophile? (A) Н2С = СН2 (B) ( СH3 )2 C = CH2

(C) Cl2C = CCl2 (D) CF3C = CH2 iii) Dự đoán sản phẩm chính của phản ứng cộng bromine dưới đây, chỉ rõ hóa lập thể:

iv) Đề xuất cấu trúc sản phẩm chính của phản ứng bromo hóa (1E)-but-1-en1-ylcyclohexane.

v) Xác định cấu trúc sản phẩm chính của phản ứng sau:

vi) Xác định sản phẩm chính của phản ứng sau:

98

Hướng dẫn i) C2H5OC2H5 ii) ( СH3 )2 C = CH2 (B) có thể phản ứng với các tác nhân electrophile nhanh nhất bởi nó có nhiều các nhóm methyl nhường electron quanh liên kết đôi. iii) Phản ứng cộng hợp electrophile vào propylene dẫn đến sự tạo thành hỗn hợp racemic của hai đối quang.

iv)

v) Phản ứng cộng nước bromine vào methylenecyclopentane là một ví dụ về phản ứng cộng electrophile vào liên kết đôi với sự tham gia của một tác nhân nucleophile “bên ngoài”. Phản ứng này được gọi là phản ứng cộng “liên hợp”. Trong phản ứng của một alkene không đối xứng, halogen luôn được gắn vào nguyên tử carbon có số hydrogen lớn nhất, còn tác nhân nucleophile (nước) vào nguyên tử carbon với số hydrogen nhỏ hơn.

99

vi) Vi hạt có tính electrophile là I+ do nguyên tử chlorine âm điện hơn. Tác nhân electrophile sẽ tác kích vào liên kết đôi để tạo thành carbocation bậc hai bền hơn.

100

5. Phản ứng cộng electrophile i) Tại sao phản ứng sau lại tạo thành 1,2-dibromocyclopentane ở dạng hỗn hợp tiêu triền (racemic)?

ii) Sản phẩm nào không được tạo thành trong phản ứng dưới đây:

iii) Dự đoán sản phẩm chính của các phản ứng sau: a)

b)

c)

d)

e)

f)

g)

101

iv) Xác định các tác nhân và điều kiện phù hợp với mỗi chuyển hóa sau của 1-methylcyclohex-1-ene?

(A) H2O, 60%H2SO4 , 0 − 10 oC (B) H2 , PtO2

(C) 1.Hg (OCOCH3 )2 ,THF,CH3OH; 2.NaBH4 ,CH3OH (D) 1.BH3 ,THF; 2.H2O2 ,NaOH,H2O (E) HBr,h, (PhCOO)2 (F) HBr,CH3COOH (G) Br2 ,CCl4 ,0 − ( −5 ) C o

102

Hướng dẫn i) Sơ đồ phản ứng bromine hóa cyclopentene tạo thành 1,2-dibromocyl clopentane:

ii) Các hợp chất C và D không được tạo thành trong phản ứng của 1methylcyclopentene với nước bromine. Electrophile trong phản ứng này là Br+, còn nucleophile là Br- và H2O. iii) a) Đây là phản ứng cộng electrophile, diễn ra qua trung gian carbocation bền hơn.

b) Trong trường hợp này nucleophile là nước. Phản ứng không sử dụng sulfuric acid đặc nên khó xảy ra sự tách nước.

c) Phản ứng hydrobor hóa-oxi hóa (cộng hợp phản Markovnikov):

d) Do nguyên tử chlorine âm điện hơn nên nguyên tử bromine thể hiện các tính chất electrophile. Sự tác kích diễn ra bởi anion chloride ưu tiên diễn ra ở nguyên tử carbon có xu hướng tạo thành carbocation bậc ba bền hơn. 103

e) Acid sẽ proton hóa liên kết đôi có nhiều nhóm thế hơn, dẫn đến sự tạo thành carbocation bền hơn.

f) Nucleophile là methanol.

g)

iv)

104

6. Quy tắc Markovnikov Năm 2018 đã diễn ra lễ kỉ niệm 180 năm ngày sinh nhà Hóa học vĩ đại Vladimir Vasilievich Markovnikov. Markovnikov đã dành sự nghiệp khoa học của mình để nghiên cứu về các mô hình cộng hợp các hợp chất khác nhau vào liên kết đôi C=C, bao gồm hợp chất A (ω(H) = 0.788%). Ví dụ, ông đã nhận thấy rằng tương tác của hợp chất A với hydrocarbon B (ω(C) = 85.71%; khi oxi hóa với potassium permanganate trong điều kiện khắc nghiệt sẽ tạo thành sản phẩm hữu cơ duy nhất C, đi kèm với sự giải phóng CO2) sẽ tạo thành hợp chất X. Dựa trên phản ứng này, Markovnikov đã đưa ra một nhận xét mà về sau trở nên nổi tiếng với tên gọi “quy tắc Markovnikov”, và sản phẩm tạo thành trong phản ứng này được gọi là Markovnikovsky. 1) Phát biểu quy tắc Markovnikov. 2) Xác định các hợp chất A-C và X, biết rằng C không chứa các nguyên tử carbon bậc ba và để trung hòa C cần 17.5 mL dung dịch NaOH 0.500 M. Thay vì hợp chất A, việc sử dụng các hợp chất khác có liên quan, ví dụ như D (ω (H) = 2.76%) khiến Markovnikov gặp một vài khó khăn. Về sau các nhà nghiên cứu nhận ra rằng tương tác của B với D tạo thành sản phẩm Markovnikovsky E diễn ra trong khoảng 6-7 tháng. Để tăng tốc độ phản ứng này, vào năm 1990, các nhà khoa học Hoa Kì đã đề xuất sử dụng hợp chất F (ω(O) = 13.45%) cùng với oxide ướt G (ω (O) = 47.07%) làm tác nhân phản ứng. 3) Xác định các hợp chất D-G. Xử lí 3-chloropropionic acid với SOCl2, sau đó cho sản phẩm H phản ứng với alkene đơn giản nhất I khi có mặt AlCl3 khan. Hợp chất đối xứng J tạo thành được chưng cất trên sodium carbonate khan, tạo thành sản phẩm không vòng đối xứng K. Dưới tác động của khí A, thay vì tạo ra sản phẩm Markovnikow như dự đoán thì lại thu được hợp chất L chỉ chứa C, H, O (trong trường hợp này, các nguyên tử carbon trong L tạo thành một vòng năm cạnh). Nếu phản ứng tiếp tục thì khí A dư sẽ từ từ cộng hợp vào L và tạo thành sản phẩm hữu cơ duy nhất M.

4) Xác định cấu trúc các hợp chất H-M. Giải thích tại sao khi A cộng hợp vào L thì chỉ tạo thành duy nhất một sản phẩm hữu cơ, mặc dù trong

105

trường hợp này, quy tắc Markovnikov không thể đưa ra một câu trả lời rõ ràng cho việc sản phẩm cộng nào sẽ chiếm ưu thế. Hướng dẫn 1) Theo Bách khoa toàn thư Hóa học (Chemical Encyclopedia), quy tắc Markovnikov được phát biểu như sau: Khi các protic acid (acid phân li H+) hoặc nước cộng hợp vào một alkene hoặc alkyne bất đối xứng, thì nguyên tử hydrogen sẽ gắn vào nguyên tử carbon có nhiều hydrogen hơn. 2) Nếu giả sử rằng hợp chất A chứa một nguyên tử hydrogen thì khối lượng mol của nó là M(A) = 1.01 / 0.00788 = 128.2 g / mol, rất dễ để đoán được A chính là HI. Tiếp theo, ta xác định công thức tối giản của hydrocarbon B. Với B thì số mol carbon (C) : (H) = 85.71 / 12.01: 14.29 / 1.01 = 7.137: 14.15 = 1: 1.98 ≈ 1: 2. Do đó, công thức của B là CnH2n. Do đề bài liên quan đến quy tắc Markovnikov, rõ ràng B là một alkene, không thể là cycloalkane. Khi alkene bị oxi hóa bởi potassium permanganate trong điều kiện khắc nghiệt thì xảy ra sự phá vỡ liên kết C=C, tạo thành ketone, carboxylic acid hoặc CO2 - tùy thuộc vào cấu trúc của alkene. Trong trường hợp này, CO2 được giải phóng trong phản ứng oxi hóa alkene với liên kết đôi ở đầu mạch. Do C thể hiện tính acid nên nó là carboxylic acid và là acid đơn chức, do dicarboxylic acid chỉ có thể được tạo thành trong phản ứng oxi hóa cycloalkene hoặc diene, chứ không phải alkene. Từ kết quả chuẩn độ, có thể xác định số mol acid ban đầu: (C) = (NaOH) = C(NaOH) · V (NaOH) = 0.50 · 0.0175 = 0.00875 mol. Khi đó M(C) = 0.77 / 0.00875 = 88 g/mol, tương ứng với công thức phân tử C4H8O2. Do C không chứa các nguyên tử carbon bậc ba nên nó là butyric acid (butanoicacid, và hydrocarbon B là pent-1-ene. Khi đó, X là 2-iodopentane.

3) Tương tự như trường hợp xác định chất A, M(D) = 1.01 / 0.0276 = 36.6 g / mol, tương ứng với hydrogen chloride HCl. Thực tế, phản ứng của B với HCl cần thời gian tương đối lâu, bởi liên kết H-Cl mạnh hơn liên kế H-I (do đó trong thực tế thì HI thể hiện tính acid mạnh hơn HCl). Phản ứng tạo thành 2chloropentane E:

106

Bây giờ, chúng ta xác định các hợp chất F và G. Để tính được thành phần oxide G, công thức của nó có thể được biểu diễn là Y2Ox. Hàm lượng của oxygen được cho trong đề bài có thể được biểu diễn theo ω(O) = 0.4707 = 16.00 · x / (16.00 · x + 2y), trong đó y là khối lượng mol của nguyên tố Y. Có thể dễ dàng tính được y = 9x và chỉ có trường hợp Y là Al là phù hợp. Vậy công thức oxide G là Al2O3. Giả sử rằng F chứa 1 nguyên tử oxygen, khi đó M (F) = 16.00 / 0.1345 = 119.0 g / mol. Rõ ràng, chất F phải chứa chlorine, và do khối lượng mol của F là số nguyên nên F phải chứa số nguyên tử chlorine chẵn. Nếu số nguyên tử chlorine trong F là 2 thì phần còn lại của các nguyên tố (ngoại trừ O và Cl) là 119 - (71 + 16) = 32 g / mol - tương đương với 1 nguyên tử lưu huỳnh. Do đó F là SOCl2. Phản ứng cộng HCl bởi tác động của SOCl2 và Al2O3 được giải thích bởi thực tế rằng việc sử dụng Al2O3 ướt dẫn đến sự thủy phân trực tiếp SOCl2 trong hỗn hợp phản ứng, dẫn đến sự tạo thành HCl. Sự gia tăng tốc độ phản ứng trong điều kiện này được giả định rằng liên kết H-Cl bị suy yếu bởi sự tạo thành liên kết hydrogen với bề mặt aluminum oxide. 4) Khi xử lí 3-chloropropionic acid với SOCl2, thì tạo thành acid chloride H, chất này cũng có thể được xác định bởi hàm lượng chlorine. Alkene đơn giản nhất là ethylene (I). Giả sử hợp chất J chứa một nguyên tử chlorine, thì M(J) = 35.45 / 0.4574 = 77.50 g / mol. Các nguyên tố có thể có còn lại (C, H, O) chiếm 77.50 - 35.45 = 42.05 g / mol - giá trị này có thể tương ứng với C3H6 hoặc C2H2O. Tuy nhiên, không có phương án nào trong số này phù hợp (ít nhất là bởi nếu có một nguyên tử chlorine thì J phải chứa số nguyên tử hydrogen lẻ). Nếu J chứa hai nguyên tử chlorine thì M(J) = 35.45 · 2 / 0.4574 = 155.0 g / mol. Các nguyên tố có thể có còn lại (C, H, O) chiếm 155.0 - 35.45 · 2 = 84.11 g / mol - giá trị này có thể tương ứng với C5H8O (biến thể với 5 nguyên tử carbon là hợp lí, do các chất phản ứng lần lượt chứa 3 và 2 nguyên tử carbon). Do đó, công thức phân tử của J là C5H8Cl2O, bằng đúng tổng công thức phân tử của các tác nhân. Do đó, trong phản ứng, acid chloride H đã cộng hợp qua liên kết C=C của ethylene, tạo thành 1.5-dichloropentan-3-one (J) đối xứng.

107

Khi xử lí với base (Na2CO3), đun nóng dẫn đến sự phân cắt hai phân tử HCl, tạo thành penta-1.4-diene-3-one K (cũng có thể dự đoán được sự hiện diện của các liên kết C=C trong K trên cơ sở rằng phản ứng của K với HI có thể tạo thành sản phẩm Markovnikov). Bây giờ, hãy thiết lập công thức phân tử của L. Nếu L chứa một nguyên tử oxygen, thì M(L) = 16.00 / 0.1949 = 82.09 g / mol, tương ứng với công thức C5H6O. Hợp chất K có cùng công thức phân tử, vậy là dưới tác động của HI, nó vòng hóa để tạo thành cyclopent-2-enon L theo cơ chế dưới đây (phản ứng này được gọi là vòng hóa Nazarov). Cũng cần chú ý rằng cấu trúc L cũng có thể được thiết lập mà không cần xét đến cơ chế tạo thành nó, do với hợp chất có vòng carbon năm cạnh có công thức phân tử C5H6O, chúng ta chỉ có thể đề xuất duy nhất một cấu trúc mà phản ứng cộng HI trong giai đoạn tiếp theo có thể được chọn lọc. L phản ứng với HI dư, chỉ tạo thành sản phẩm 3-iodocyclopentanone M. Sự chọn lọc vị trí của phản ứng trong trường hợp này được giải thích bởi các tính chất hút electron mạnh của nhóm carbonyl.

108

7. Cộng electrophile 1) Xác định sản phẩm tạo thành khi cộng hợp ClCH=CHOC2H5 với HCl (a) và Br2 khi có mặt LiBr* đánh dấu đồng vị (b). 2) Xác định sản phẩm tạo thành khi cộng hợp HCl (a) và Br2 khi có mặt LiBr* đánh dấu đồng vị (b) với: CF3CH = CHCH2 ( CH3 )2 CH3CH = CHCOOH

Hướng dẫn 1)

2)

109

A B 

8. Cộng electrophile 1) Xác định sản phẩm được tạo thành trong phản ứng cộng ICl (a) và Hg(OAc)2 trong methanol (sau đó demercury hóa) (b) của các hợp chất sau:

CH2 = CHCOOEt  A 

1-phenylcyclohexene B

2) Xác định sản phẩm được tạo thành trong phản ứng cộng HCl (a) và Cl2 trong nước (b) của các hợp chất sau:

N  CCH = CHOCH3  A 

EtCH = CHPh B

Hướng dẫn 1)

2)

110

9. Đề xuất cơ chế phản ứng 1) Xử lí pent-4-ene-1-ol với bromine trong carbon tetrachloride thu được một hợp chất có công thức phân tử C5H9OBr. Biểu diễn cấu tạo sản phẩm và đề xuất cơ chế khả dĩ cho chuyển hóa này. 2) Dự đoán cấu tạo sản phẩm A và đề xuất một cơ chế khả dĩ cho quá trình tạo thành sản phẩm này:

A có thể có bao nhiêu đồng phân lập thể? 3) Khi xử lí heptadiene-1,6 với nước bromine thì nhận được một hợp chất có cấu tạo như sau:

Hãy đề xuất cơ chế tạo thành hợp chất này.

111

Hướng dẫn 1)

2)

A có hai nguyên tử carbon bất đối nên có thể có bốn đồng phân lập thể. 3)

112

10. Chuyển vị carbocation i) Xác định công thức cấu tạo sản phẩm cộng nước của tert-butylethylene và isopropylethylene trong môi trường acid. Trình bày cơ chế tạo thành các sản phẩm đó.

ii) Xác định sản phẩm chính và đề xuất cơ chế của phản ứng:

Hướng dẫn i)

113

ii) Chuyển hóa của 2,2-dimethylcyclohexanol trong môi trường acid đi kèm với chuyển vị như sau:

114

11. Chuyển vị carbocation Hợp chất 2,2,4-trimetylpentan (A) được sản xuất với quy mô lớn bằng phương pháp tổng hợp xúc tác từ C4H8 (X) với C4H10 (Y). A cũng có thể được điều chế từ X theo hai bước: thứ nhất, khi có xúc tác axit vô cơ, X tạo thành Z và Q; thứ hai hydro hóa Q và Z a) Viết các phương trình phản ứng để minh họa và gọi tên các hợp chất X, Y, Z, Q theo danh pháp IUPAC b) Ozon phân Z và Q sẽ tạo thành 4 hợp chất, trong đó có axeton và fomandehit, viết cơ chế phản ứng Hướng dẫn a)

Bước thứ nhất gồm tương tác giữa hai phân tử trong môi trường axit

115

b)

116

12. Tổng hợp linalool Linalool - một hợp chất tạo hương phổ biến, được dùng trong nhiều loại mỹ phẩm: xà phòng, nước hoa, dầu gội đầu, … Các nhà nghiên cứu ước lượng rằng khoảng 60-80% các loại sản phẩm như trên có chứa hợp chất hữu cơ này, do đó việc tổng hợp thương mại linalool đóng vai trò quan trọng để đáp ứng nhu cầu của thị trường, thay cho nguồn linalool tự nhiên khan hiếm. 1) Vẽ cấu trúc 3,7-dimethylocta-1,6-dien-3-ol (linalool). 2) Dùng dấu hoa thị để đánh dấu các nguyên tử carbon thủ tính. Dưới đây là một sơ đồ tổng hợp linalool:

3) Xác định các sản phẩm khác có thể được tạo thành trong phản ứng cộng của HBr vào isoprene. Vẽ cấu trúc của chúng. 4) Đề xuất các tác nhân B và C phù hợp với sơ đồ trên. 5) Đề xuất cơ chế phản ứng giữa A và alkene. Một người học hóa có cái nhìn tốt về cấu trúc linalool có thể nhận thấy rằng hợp chất này có thể chuyển thành vòng 6 cạnh trong các điều kiện phù hợp. Mọi người đều biết rằng các tình nguyện viên tổ chức Olympiad khoa học Lithuania (thường được gọi là các oranžiniai - hay người da cam) thực sự rất thích cam. Cam có mùi dễ chịu bởi limonene - một sản phẩm vòng hóa của linalool. 6) Đề xuất điều kiện phản ứng (trung tính/acid/base), tác nhân và cơ chế cho chuyển hóa này.

117

Hướng dẫn 1-2)

3)

4)

5)

6) Môi trường có tính acid sẽ phù hợp. H2SO4 có thể đóng vai trò là một acid lẫn base yếu (anion) để có thể thực hiện deproton hóa trong bước cuối của cơ chế. Cơ chế đề xuất:

118

13. Chuyển vị carbocation Đề xuất cơ chế phản ứng:

Hướng dẫn

119

14. Hydrobor hóa alkene i) Alcohol nào được tạo thành trong phản ứng hydrobor hóa methylcyclohexene?

1-

ii) Đề xuất cơ chế cho phản ứng ở ý i. iii) Xác định sản phẩm chính của phản ứng sau và chỉ rõ hóa lập thể:

Hướng dẫn 1) Sản phẩm tạo thành là B (2-methylcyclohexanol) ở dạng hỗn hợp tiêu triền.

ii) Phản ứng hydrobor hóa 1-methylcyclohexene, sau đó xử lí hỗn hợp phản ứng với H2O2 /NaOH/H2O là phương pháp để tạp thành các alcohol từ alkene. Giai đoạn đầu tiên của phản ứng hydrobor hóa là quá trình cộng hợp phản Markovnikov, nghĩa là boron được gắn vào nguyên tử carbon ít nhóm thế hơn. Sự oxi hóa liên kết C-B tạo thành alcohol.

120

iii)

121

15. Oxi hóa alkene i) Xác định sản phẩm chính của phản ứng giữa 1,2-dimethylcyclohexene với meta-chloroperbenzoic acid.

ii) Xác định hóa lập thể của 1-phenylpropane-1,2-diol trong phản ứng:

(A) (1R, 2R) (B) (1S, 2S) (C) (1R, 2S) (D) (1S, 2R) (E) Hỗn hợp đẳng lượng của (1R, 2R) và (1S, 2S) (F) Hỗn hợp đẳng lượng của (1R, 2S) và (1S, 2R) (G) Hỗn hợp đẳng lượng của (1R, 2R) và (1R, 2S) (H) Hỗn hợp đẳng lượng của (1S, 2S) và (1S, 2R) iii) Xác định công thức chất đầu A của phản ứng:

iv) Hãy đề xuất chuyển hóa trans-1,2-diphenylethylene (trans-stilbene) thành meso- và d, l-1,2-diphenyl-ethanediol-1,2. Vẽ công thức chiếu Fischer của các diol tạo thành (mạch carbon xếp dọc) và dẫn ra kí hiệu R, S cho các tâm bất đối.

122

Hướng dẫn i)

ii) Phản ứng oxi hóa (1E)-prop-1-en-1-ylbenzene bởi KMnO4 tạo thành hỗn hợp đẳng mol các đồng phân (1R, 2R) và (1S, 2S) của 1-phenylpropane-1,2diol.

iii) A là 1,2-dimethylcyclopentene.

iv) Có thể sử dụng KMnO4 hoặc OsO4 để chuyển hóa convert trans-1,2diphenylethylene (trans-stilbene) thành meso-1,2-diphenylethanediol-1,2. d,l-1,2-Diphenylethanediol-1,2 có thể được tạo thành bởi phản ứng của trans1,2-diphenylethylene với meta-chloroperbenzoic acid hoặc các peracid khác, sau đó xử lí với dung dịch kiềm.

123

124

16. Phản ứng ozone phân và oxi hóa phân cắt i) Xác định công thức alkene mà khi ozone phân sẽ tạo thành acetone và methyl ethyl ketone. ii) Hợp chất nào không được tạo thành trong phản ứng sau:

iii) Xác định công thức các sản phẩm chính A – H nhận được từ mỗi phản ứng sau:

iv) Xác định sản phẩm chính của phản ứng sau:

125

v) Ozone phân hợp chất C6H12 chỉ tạo thành duy nhất một sản phẩm C3H6O. Phản ứng bromo hóa gốc tự do C6H12 chỉ tạo thành duy nhất một sản phẩm C6H11Br. Hãy xác định công thức chất C6H12 ban đầu và viết tất cả các phương trình phản ứng. Hướng dẫn i) Hợp chất đó là 2,3-dimethylpent-2-ene:

ii) Phản ứng không tạo thành hợp chất D.

iii)

126

iv)

127

v) Hợp chất C6H12 ban đầu là 2,3-dimethylbut-2-ene (tetramethylethylene).

128

17. Phản ứng của epoxi Dự đoán cấu trúc các hợp chất A, B tạo thành trong các chuyển hóa sau đây. Với hợp chất B, hãy dẫn ra công thức chiếu Fischer và kí hiệu các tâm bất đối theo hệ thống R, S. Cho biết B là đồng phân erythro- hay threo-?

Hướng dẫn Tạo ra oxirane từ bromohydrin:

Mở vòng oxirane tạo thành các glycol:

129

18. Chuyển hóa hữu cơ Sử dụng bất kì tác nhân cần thiết nào khác, hãy đề xuất cách chuyển hóa methylenecyclopentane thành các hợp chất sau:

Hướng dẫn

130

19. Xác định hydrocarbon Có 6 alkene vòng, không chứa nhóm ethyl, là đồng phân, có cùng công thức C5H8. 3 trong số 6 hợp chất trên được cho vào các bình dán nhãn A, B, C nhưng bạn không biết hợp chất trong mỗi bình là gì. Dựa vào kết quả của các phản ứng với KMnO4 sau, hãy xác định công thức của các chất A-F. -

Hợp chất A tạo thành dicarboxylic acid D chứa một nguyên tử carbon thủ tính.

-

Hợp chất B tạo thành ketone E không chứa nguyên tử carbon thủ tính nào.

-

Hợp chất C tạo thành F chứa cả nhóm chức carboxylic acid và ketone và cũng có một nguyên tử carbon thủ tính.

Hướng dẫn

131

20. Xác định hydrocarbon Hydrocarbon C5H10 [A] tham gia vào các phản ứng sau: +

KMnO4 /H A ⎯⎯⎯⎯ → Hai acid Br2 /CCl4 A ⎯⎯⎯ → B (dạng threo) 1. OsO4 A ⎯⎯⎯⎯⎯ → C (dạng erythro) 2. NaHSO3 /H2O

Xác định cấu trúc A, B, C và biểu diễn các cấu trúc của B và C theo công thức chiếu Fischer. Viết các phương trình phản ứng. Hướng dẫn

132

21. Xác định hydrocarbon Hydrocarbon C8H16 [A] tham gia vào các phản ứng sau: 1.O3 A ⎯⎯⎯ → B (ketone duy nhất) 2.Ph3P Br2 A ⎯⎯→ C (dạng meso) CCl4 1.BH3 A ⎯⎯⎯⎯ → D (racemate) 2.H O ,OH− 2 2

Xác định cấu trúc A - D và biểu diễn các cấu trúc của C và D theo công thức chiếu Fischer. Viết các phương trình phản ứng. Hướng dẫn

133

22. Sơ đồ chuyển hóa 1) Hoàn thành sơ đồ chuyển hóa sau:

2) Hydrocarbon C10H20 (A) tham gia vào các chuyển hóa sau: 1.O3 A ⎯⎯⎯⎯⎯ → 2 ( CH3 )2 CHC ( O) CH3 2.Zn/CH3COOH 1.RCOOOH A ⎯⎯⎯⎯ → B (dạng meso) 2.H O/OH− 2

Br2 A ⎯⎯ → C (dạng meso) 1.BH3 A ⎯⎯⎯⎯ → D (racemate) 2.H O /OH− 2 2

Xác định cấu trúc A, B, C, D và biểu diễn các cấu trúc của B, C và D theo công thức chiếu Fischer. Viết các phương trình phản ứng.

134

Hướng dẫn 1)

2)

135

136

23. Alkyne i) Từ acetylene, 1-bromopropane và các tác nhân vô cơ cần thiết khác, hãy đề xuất cách điều chế 1-bromopentane. ii) Có thể nhận được tert-butyl acetylene bởi phản ứng giữa tert-butyl bromide với sodium acetylenide không? iii) Xác định các sản phẩm phản ứng a-k:

KAPA là muối potassium của propane-1,3-diamine. iv) Đề xuất quy trình tổng hợp oxirane A từ ethylacetylene và 1-propane iodine cùng các tác nhân vô cơ cần thiết khác. Gọi tên hợp chất A theo danh pháp R, S.

137

Hướng dẫn i) Giai đoạn đầu tiên là tạo thành anion acetylenide. Tiếp đó là phản ứng của 1-bromopropane với anion acetylenide, diễn ra theo cơ chế SN2:

ii) Không thể nhận được tert-butyl acetylene bởi sodium acetylenide sẽ đóng vai trò như một base, gây ra phản ứng tách, tạo thành sản phẩm là isobutylene. iii) a) Phản ứng này là một phương pháp để điều chế các cis-alkene:

b) Phản ứng Kucherov, một phương pháp để tổng hợp các ketone:

c) Chuyển hóa hydrobor hóa – oxi hóa, một hướng để tạo ra các aldehyde:

d) Phản ứng này là phương pháp để tổng hợp các trans-alkene. Sự tạo thành các trans-alkene được giải thích bởi trung gian anion-gốc trans-vinyl bền hơn.

e) Một phương pháp khác để điều chế các cis-alkene nhờ phản ứng cộng hợp syn của borane:

138

f) Phản ứng này thể hiện sự ưu tiên vị trí của phản ứng cộng hợp disiamylborane cồng kềnh (bị án ngữ không gian) vào liên kết ba:

g) Một ví dụ về tổng hợp các acetylene alcohol:

h) Phản ứng đồng phân hóa của một alkyne giữa mạch thành một alkyne đầu mạch kém bền về mặt nhiệt động học hơn:

iv) A có thể được điều chế dưới dạng hỗn hợp các đối quang:

139

Quy trình:

140

24. Alkyne i) Từ acetylene và 1-bromopropane, hãy đề xuất quy trình điều chế pentanal và pentan-2-one với yêu cầu là trong cả hai trường hợp thì ở giai đoạn cuối, chất đầu phản ứng phải giống nhau. ii) Chọn ra các tác nhân phù hợp nhất dưới đây để chuyển hóa 3,3dimethylbut-1-yne thành 3,3-dimethylbutanal. (A) Н2О, HgSO4 ( xúc tác ) , H2SO4 (B) 1. Sia2BH, THF, 0o C; 2. H2O2 , NaOH, H2O (C) KMnO4 , NaOH, H2O (D) 1. HOBr; 2. NaOH, H2O iii) Hãy chọn ra điều kiện phù hợp nhất để chuyển hex-3-yne thành cis-hex3-ene. (A) Xúc tác Pt catalyst, H2 (B) Xúc tác Lindlar, H2 (C) Na, NH3(l) (D) NaNH2, NH3 (l) iv) Khi xử lí với AgNO3 /NH3 / C2H5OH thì hydrocarbon C6H10 tạo thành một muối bạc không tan. Oxi hóa hydrocarbon này bởi KMnO4 /H2O / to tạo thành carboxylic C5H10O2. Phản ứng với H2O /HgSO4 /H2SO4 dẫn đến sự tạo thành một ketone C6H12O. Xác định cấu tạo của hydrocarbon C6H10 này, biết rằng trong phổ 1H NMR cuả cả acid và ketone đều chỉ quan sát thấy hai mũi đơn của các proton. v) Hoàn thành sơ đồ chuyển hóa dưới đây:

141

vi) Hoàn thành sơ đồ tổng hợp dodeca-5,7-diyne B và hợp chất C.

vii) Đề xuất quy trình tổng hợp (5E, 7Z)-dodeca-5,7-diene từ hex-1-yne và các tác nhân phổ biến sẵn có. Hướng dẫn i)

142

ii)

iii) Điều kiện phù hợp nhất là sử dụng H2 với xúc tác Lindlar (B). iv) Hydrocarbon đó là 3,3-dimethylbut-1-yne.

v)

143

vi)

vii) Có thể thực hiện tổng hợp (5E, 7Z)-dodeca-5,7-diene từ hex-1-yne bởi các phản ứng ghép cặp chéo.

144

145

25. Tổng hợp cyclopropane Từ propylene, 1-bromopropane, CH2I2 và các tác nhân vô cơ cần thiết, hãy đề xuất quy trình tổng hợp cis-1-methyl-2-propylcyclopropane. Hướng dẫn

Phản ứng cyclopropane hóa cũng có thể được thực hiện bởi tác động của CH2N2 / Pd (OAc )2 / Et2O .

146

26. Hệ liên hợp i) Hợp chất nào sau đây có nhiệt hydrogen hóa cao hơn?

ii) Cấu trúc nào sau đây có chứa hệ liên hợp?

iii) Phản ứng cộng 1 đương lượng bromine vào hexa-2,4-diene ở 0 oC tạo thành 4,5-dibromohex-2-ene (A) và đồng phân B. Đồng phân B là (A) 5,5-Dibromhex-2-ene (B) 2,5-Dibromhex-3-ene (C) 2,2-Dibromhex-3-ene (D) 2,3-Dibromhex-3-ene iv) Sau phản ứng của khí HCl với 2-methylbuta-1,3-diene, phân lập được bốn hợp chất mới (dữ kiện từ sắc kí khí-lỏng). Hãy xác định cấu tạo các hợp chất này. v) Xác định các hợp chất A – G:

vi) Xác định cấu tạo các sản phẩm chính của phản ứng cộng 1,2- và 1,4- của khí hydrogen chloride vào 3-methylenecyclohexene và chỉ ra trong số chúng sản phẩm nhiệt động học và sản phẩm động học.

147

vii) Giải thích sự tạo thành các hợp chất A – C trong phản ứng:

Hướng dẫn i) Do trong A có chứa hệ liên hợp 1,3-diene nên chất này bền hơn, do đó chất B (kém bền hơn) sẽ có nhiệt hydrogen hóa cao hơn. ii) Các cấu trúc C và D có chứa hệ liên hợp. Trong cấu trúc A, nguyên tử carbon mang điện tích dương và liên kết đôi bị tách ra bởi một nguyên tử carbon lai hóa sp3, điện tích không thể được giải tỏa, do đó không có sự liên hợp. Trong cấu trúc B, liên kết đôi và nhóm carbonyl cũng bị tách ra bởi một nguyên tử carbon lai hóa sp3. iii) 2,5-dibromohex-3-ene (B). iv) Sơ đồ dưới đây giải thích cho sự tạo thành bốn sản phẩm mới A – D (dữ kiện GLC) trong phản ứng của khí HCl với 2-methylbuta-1,3-diene. Sự proton hóa của các liên kết đôi thứ nhất và thứ hai tạo thành các carbocation allyl được bền hóa cộng hưởng. Do đó, sự tác kích của ion chloride dẫn đến sự tạo thành bốn alkyl halide đồng phân, trong đó chiếm hàm lượng chủ yếu là các đồng phân C và D.

148

v)

vi) Trước tiên, cần xác định được nguyên tử carbon lai hóa sp2 nào của hệ liên hợp là carbon C1. Proton sẽ gắn vào nguyên tử đã được xác định, do trong trường hợp này carbocation tạo thành bền hơn carbocation nhận được khi proton gắn vào các phần khác của hệ liên hợp. Sản phẩm động học là 3methyl-3-chlorocyclohexene, trong khi đó 1-methyl-3-chlorocyclohexene là sản phẩm nhiệt động học – bởi nó chứa liên kết đôi nhiều nhóm thế hơn.

vii) Giải thích tạo thành các hợp chất A – C trong phản ứng của HCl với 1methylcycloocta 1,3-diene: i) Phản ứng cộng có thể diễn ra ở liên kết đôi C1-C2 nhiều nhóm thế hơn, dẫn đến sự tạo thành một carbocation bậc ba. Nucleophile sẽ gắn vào carbocation này, tạo thành sản phẩm cộng 1,2.

149

ii) Phản ứng cộng có thể diễn ra ở liên kết đôi C3-C4 ít nhóm thế hơn, tạo thành một carbocation bậc hai. Nucleophile sẽ gắn vào carbocation này, tạo thành sản phẩm cộng 1,2.

iii) Phản ứng cộng có thể diễn ra ở liên kết đôi C3-C4 ít nhóm thế hơn, tạo thành một carbocation bậc hai. Dạng cộng hưởng của cation này là một cation bậc ba dạng allyl bền hơn. Nucleophile sẽ gắn vào carbocation này, tạo thành sản phẩm cộng 1,4.

150

27. Phản ứng Diels-Alder i) Xác định sản phẩm tạo thành trong phản ứng sau, chỉ rõ hóa lập thể.

ii) Xác định các diene được dùng trong các phản ứng sau:

151

iii) Hoàn thành sơ đồ phản ứng:

Hướng dẫn i) Phản ứng Diels-Alder có tính đồng bộ (toàn bộ các liên kết mới được tạo thành đồng thời) và diễn ra với đặc trưng lập thể (cộng cis vào liên kết đôi) – nghĩa là cấu hình của diene và dienophile được bảo toàn trong quá trình tạo thành sản phẩm cộng. Trong tiến trình phản ứng, có hai liên kết σ và một liên kết π mới được tạo thành.

Một cách viết khác:

152

ii) a)

b)

c)

d)

e)

f)

g)

h)

iii) Chất A là benzo-1,4-quinone.

153

28. Phản ứng Diels-Alder i) Có bao nhiêu cyclohexene đồng phân có thể được tạo thành trong phản ứng Diels-Alder của diene A và dienophile B (không kể đến các đối quang)?

ii) Xác định các sản phẩm A – D tạo thành bởi các phản ứng Diels-Alder:

154

Hướng dẫn i)

ii)

155

29. Phản ứng vòng hóa Diels-Alder 1) Từ triphenylphosphine, ethyl bromoacetate, allyl bromide và cyclopentene, cùng các tác nhân cần thiết khác, hãy điều chế hợp chất A có công thức dưới đây.

2) Từ furan, acetylene, formaldehyde và buta-1,3-diene, cùng các tác nhân cần thiết khác, hãy điều chế hợp chất B.

156

Hướng dẫn 1)

2)

157

30. Hiện tượng đồng phân lập thể của các hợp chất vòng Một ví dụ về hiện tượng đồng phân là đồng phân cis-trans trong các hợp chất vòng. Sự khám phá ra hiện tượng đồng phân này trong nhiều hợp chất tự nhiên, dược phẩm và các hợp chất có hoạt tính sinh học khác đã thúc đẩy sự phát triển của các phương pháp tổng hợp hữu cơ, dẫn đến việc tạo thành những hợp chất với sự sắp xếp phù hợp của các nhóm thế với mặt phẳng vòng. Hãy cùng xem xét các bài tập được mô tả dưới đây, nói về hiện tượng đồng phân cis-trans, và trả lời các câu hỏi.

(Kí hiệu là A1, A2, … và B1, B2, … nếu có nhiều sản phẩm) A. Hai phản ứng của 3-methylcyclohexane được biểu diễn trong sơ đồ trên. 1) Vẽ công thức cấu tạo của tất cả các sản phẩm tạo thành trong các phản ứng. Lưu ý rằng phản ứng sử dụng hỗn hợp racemic của 3methylcyclohexane. 2) Xác định cấu hình tuyệt đối của tất cả các nguyên tử carbon bất đối hiện diện trong sản phẩm. 3) Vẽ công thức dạng ghế của sản phẩm trong đó tất cả các nguyên tử carbon bất đối có cấu hình R. 4) Vẽ công thức cấu tạo của các trung gian trong cả hai phản ứng. (Bỏ qua hóa lập thể). B. Các đồng phân hình học của 3,4-dimethylcyclopentanone, C và D (chú ý: một trong chúng tồn tại ở dạng racemate) phản ứng với NaBH4 trong ethanol, và sau đó được thủy phân trong môi trường acid. Trong phổ 1H NMR của các sản phẩm tạo thành từ đồng phân C chỉ có một bộ tín hiệu, còn với các sản phẩm từ D thì quan sát thấy hai hợp chất riêng biệt. 1) Vẽ công thức cấu tạo của C và D và các sản phẩm cuối tương ứng trong các phản ứng. 2) Có bao nhiêu tín hiệu trong phổ 1H NMR của các hợp chất C và D? Đánh dấu các nhóm proton trong công thức cấu tạo của C và D có cùng độ chuyển dịch hóa học.

158

C. Các hợp chất đồng phân E và F (có công thức dưới đây), được tạo thành trong phản ứng Diels-Alder. Vẽ công thức cấu tạo các hợp chất để tổng hợp E và F.

Hướng dẫn A.1-2)

3)

4)

B.1) C là hỗn hợp racemic.

159

Phản ứng khử các đồng phân tạo thành các alcohol sau: Phản ứng khử của đồng phân D tạo thành hai hợp chất riêng biệt trong phổ 1H NMR.

Hai đồng phân dia Đồng phân C tạo thành hỗn hợp hai đối quang, có phổ 1H NMR giống nhau.

Hai đối quang (không phân biệt được trong phổ 1H NMR) 2) Các đồng phân cis và trans, đều có 4 nhóm tín hiệu trong phổ 1H NMR.

C. Tổng hợp E từ

Tổng hợp F từ

160

31. Ảnh hưởng của tính thơm 1) Từ benzene, ethanol cùng các tác nhân cần thiết khác, hãy điều chế 1bromo-2,4,6-triethylbenzene. Viết sơ đồ phản ứng của chất này với: a) potassium amide trong ammonia lỏng. b) potassium amide trong ammonia lỏng khi có mặt kim loại potassium. 2) Điều chế tetracene [A] từ benzene, 2,3-dimethylnaphthalene cùng các tác nhân cần thiết khác.

3) Từ naphthalene, propanol-1, succinic anhydride cùng các tác nhân cần thiết khác, hãy điều chế 9-(n-propyl)phenanthrene [B].

161

Hướng dẫn 1)

2)

162

3)

163

32. Cơ chế phản ứng Đề xuất cơ chế cho phản ứng sau:

Từ ethylbenzene cùng các tác nhân cần thiết khác, hãy đề xuất hướng tổng hợp A. Hướng dẫn

Quy trình tổng hợp:

164

Chương 3

Dẫn xuất halogen, oxygen và nitrogen 1. Tổng quan về phản ứng thế nucleophile 1) Xét cơ chế phản ứng của tert-butyl chloride với ion iodide:

(CH3 )3 C − Cl

+ I− → (CH3 )3 − I + Cl−

Nếu tăng gấp đôi nồng độ ion iodide thì tốc độ tạo thành tert-butyl iodide sẽ (A) tăng 2 lần. (B) tăng 4 lần. (C) giữ nguyên. (D) giảm. (E) không phải các phương án trên. 2) Hợp chất nào sau đây không tham gia vào các phản ứng thế theo cơ chế SN1 và SN2?

3) Mỗi dẫn xuất halide sau có thể tham gia phản ứng thế nucleophile theo cơ chế nào (SN1 hay SN2)? Sắp xếp các halide theo trình tự khả năng tham gia phản ứng SN1.

165

4) Sắp xếp các chloride sau theo trình tự giảm dần hoạt tính dung môi phân trong HCOOH 80%: diphenylchloromethane, tert-butyl chloride, benzyl chloride, triphenylchloromethane. 5) Xác định công thức chất phản ứng (chất nền), tác nhân, dung môi và sản phẩm trong tổng hợp 4-(cyanomethyl)cyclohexene bởi phản ứng thế nucleophile. 6) Chất nào sau đây có hoạt tính kém nhất trong phản ứng SN2?

7) Sắp xếp các hợp chất sau theo thứ tự tăng dần khả năng phản ứng với AgNO3. Giải thích.

Hướng dẫn 1) tert-Butyl chloride là dẫn xuất halide có nhiều nhóm thế cồng kềnh (quanh nhóm chloride bị án ngữ không gian) nên có xu hướng dễ tham gia phản ứng thế theo cơ chế SN1. Xét cơ chế SN1 cho phản ứng của tert-butyl chloride với ion iodide thì thấy nucleophile không tham gia vào giai đoạn tốc định nên nếu tăng gấp đôi nồng độ ion iodide thì tốc độ tạo thành tert-butyl iodide được giữ nguyên (C). 166

2) Các hợp chất C và E không tham gia vào các phản ứng thế nucleophile SN1 và SN2. Các vinyl và aryl halide không tham gia phản ứng SN2 là do sự tiếp cận của nucleophile từ hướng ngược lại bị ngăn cản bởi đám mây electron π của liên kết đôi và vòng benzene. Khi halogen liên kết với nhóm vinyl hoặc aryl thì trên nguyên tử carbon lai hóa sp2 có một điện tích dương riêng phần và liên kết C-Hal bền hơn; ngoài ra, các cation vinyl và aryl kém bền hơn các alkyl carbocation, khiến cho phản ứng theo cơ chế SN1 không thuận lợi và gần như không thể diễn ra. 3) Các allyl và benzyl chloride đều có thể phản ứng theo cả cơ chế SN1 (tạo ra carbocation bền) lẫn SN2 (trạng thái chuyển tiếp được bền hóa bởi một hệ liên hợp). Độ bền tương đối của các carbocation tương ứng với dẫn xuất halide ban đầu giảm theo trình tự F  G  D  E  A  J  C  H  I  B

4) Phản ứng dung môi phân của các chloride trong HCOOH 80% diễn ra theo cơ chế SN1. Cation triphenylmethyl là trung gian bền nhất nên chloride tương ứng có hoạt tính mạnh nhất.

5)

167

6) Hợp chất B do trường hợp này có sự án ngữ không gian lớn nhất đối với việc tiếp cận của nucleophile với tâm phản ứng (tâm electrophile). 7) Phản ứng của các chlorohydrocarbon với dung dịch AgNO3 tạo thành alcohol xảy ra theo cơ chế SN1. Giai đoạn quyết định tốc độ phản ứng SN1 là sự hình thành carbocation, do đó hợp chất nào tạo thành carbocation bền nhất thì phản ứng nhanh nhất. -

Khả năng phản ứng của A kém nhất vì carbocation không có cấu trúc phẳng (do tồn tại cấu trúc cầu nối.) Ngoài ra, cation này cũng không được bền hoá bởi sự giải toả điện tích (cộng hưởng) làm tăng sức căng của hệ quá nhiều.

-

Hợp chất C dễ phản ứng nhất do tạo thành carbocation bậc III, có thể tồn tại cấu trúc phẳng, và được bền hoá bởi sự cộng hưởng từ ba vòng thơm.

Do vậy: A < B < C.

168

2. Tính nucleophile i) Lấy ví dụ về các nucleophile chứa С-, N-, O-, S-. ii) Hãy dẫn ra định nghĩa về tính base và tính nucleophile. Trong một chu kì và một nhóm của bảng tuần hoàn, tính base và nucleophile của các hợp chất tương ứng biến đổi như thế nào? iii) Độ mạnh-yếu của nucleophile phụ thuộc vào một vài yếu tố. Hãy cho biết phát biểu nào sau đây không đúng: (A) Anion là nucleophile mạnh hơn nguyên tử trung hòa hoặc phân tử. (B) Các nguyên tử càng phân cực mạnh thì tính nucleophile càng mạnh. (C) Mức độ solvate hóa ảnh hưởng đến tính nucleophile. (D) Trong một chu kì của Bảng tuần hoàn, tính nucleophile và tính base của các hợp chất tương ứng biến đổi cùng chiều. (E) Trong một nhóm của Bảng tuần hoàn, tính nucleophile và tính base của các hợp chất tương ứng biến đổi cùng chiều. iv) Sắp xếp các anion và hợp chất trung hòa sau theo trình tự giảm dần tính nucleophile trong dung dịch nước.

v) Sắp xếp các anion và hợp chất trung hòa sau theo trình tự giảm dần tính nucleophile.

CF3CH2O−

CH3CH2O−

CH3COO−

CH3CH2S−

(CH3CH2 )2 O

CH3CH2Li

vi) Phân loại các anion và phân tử trung hòa sau thành các nhóm theo độ mạnh tương đối của tính nucleophile so với methanol.

RCOOH,Br − ,HS− ,H2O,RO− ,RCOO− ,OH− ,I− ,NH3 ,Cl− ,ROH,F − ,NC− ,RS− ,N3− Rất tốt Tốt Chấp nhận được Yếu Rất yếu vii) Tính nucleophile của các tiểu phân trong dãy I-, Br-, Cl-, F- biến đổi như thế nào từ các dung môi aprotic sang dung môi protic?

169

Hướng dẫn i)

ii) Tính base là khả năng phản ứng của một phân tử hoặc ion với proton hoặc các Brønsted acid, còn tính nucleophile là khả năng phản ứng với carbon có tính electrophile, nghĩa là với carbocation (sp2) hoặc với nguyên tử carbon lai hóa sp3 mang điện tích dương riêng phần. Trong một nhóm của bảng tuần hoàn, theo chiều từ trên xuống, tính nucleophile của các hợp chất tương ứng sẽ tăng, trong khi đó tính base sẽ giảm (với các phản ứng trong dung môi protic). Còn trong một chu kì, tính nucleophile và tính base đều tăng theo chiều từ phải sang trái. iii) Phát biểu e sai. Trong một nhóm của Bảng tuần hoàn, tính nucleophile và tính base của các hợp chất tương ứng biến đổi ngược chiều. iv) Nucleophile yếu nhất là methanol trung hòa. Các nucleophile chứa oxygen được sắp xếp theo độ mạnh của các acid liên hợp (pKa: HOH (15.7), phenol (10), acetic acid (4.76)). Do lưu huỳnh có kích thước lớn hơn oxygen nên CH3S- > HO-. v) CH3CH2Li  CH3CH2S−  CH3CH2O−  CF3CH2O−  CH3COO−  (CH3CH2 )2 O vi) Rất tốt

I− ,HS− ,RS−

Tốt

Br − ,HO− ,RO− ,NC− ,N3−

Chấp nhận được

NH3 ,Cl− ,F − ,RCOO−

Yếu

H2O,ROH

Rất yếu

RCOOH

170

vii) Trong các dung môi aprotic (ví dụ như dimethylformamide, acetonitrile, acetone, tetrahydrofuran, dimethyl sulfoxide, hexamethyl phosphotriamide) thì tính nucleophile biến đổi như sau: I- < Br- < Cl- < F-. Trong các dung môi này, các nucleophile không bị solvate hóa mạnh như trong các dung môi protic. Sự tiêu hao năng lượng để khử-solvate hóa biến đổi trong khoảng hẹp. Hoạt tính của các nucleophile chủ yếu được quyết định năng lượng thu được khi hình thành liên kết giữa nucleophile và nguyên tử carbon của chất phản ứng, nghĩa là ái lực với tâm electrophile này – cũng giống như ái lực đối với proton, đạt giá trị lớn nhất trong trường hợp fluorine. Trong các dung môi protic (ví dụ như nước, formic acid, methanol, ethanol, acetic acid): I- > Br- > Cl- > F-. Trong các dung môi protic, tác nhân nucleophile được solvate hóa mạnh hơn nhiều so với trong các dung môi aprotic, do các liên kết hydrogen xuất hiện giữa nucleophile và dung môi. Sự tiêu hao năng lượng để khử-solvate hóa của nucleophile có thể so sánh với năng lượng thu được từ việc tạo thành liên kết halogen-carbon mới và phụ thuộc vào khả năng phân cực (bán kính) của ion halide. Do đó, trong các dung môi aprotic, tính nucleophile của ion halide tỉ lệ thuận với tính base của chúng, còn trong dung môi protic thì chủ yếu được xác định bởi khả năng phân cực của chúng.

171

3. Ảnh hưởng của dung môi i) Phát biểu nào sau đây về ảnh hưởng của dung môi trong các phản ứng thế nucleophile là không chính xác: (A) Các dung môi phân cực với hằng số điện môi lớn tạo thuận lợi cho phản ứng SN1. (B) Các dung môi aprotic phân cực tạo thuận lợi cho phản ứng SN2. (C) Các alkyl halide vẫn có thể tham gia phản ứng thế SN2 trong các dung môi phân cực với hằng số điện môi lớn. (D) Các alkyl halide bậc ba vẫn có thể tham gia phản ứng SN1 trong các dung môi không phân cực. (E) Sự chọn lọc lập thể của các phản ứng với sự tham gia của alkyl halide bậc hai không thể bị tác động bởi dung môi. ii) Giải thích xu hướng biến đổi giá trị tốc độ tương đối của phản ứng giữa n-butyl bromide với sodium azide trong các dung môi khác nhau. CH3CH2CH2CH2Br + NaN3 → CH3CH2CH2CH2N3 + NaBr Dung môi Tốc độ tương đối

CH3OH

H 2O

DMSO

DMFA

CH3CN

[(CH3)2N]3P=O

1

7

1300

2800

5000

200000

Cho biết:

iii) Giải thích ảnh hưởng của dung môi đến các phản ứng bậc hai sau: a) Tốc độ phản ứng tương đối trong H2O / trong CH3OH / trong C2H5OH = 1/16/44. 131 −

I + CH3I → CH3131I + I−

b) Tốc độ phản ứng tương đối trong hexane / trong chloroform = 1/13000.

(CH3CH2CH2 )3 N + CH3I → (CH3CH2CH2 )3 N+ (CH3 )I−

172

Hướng dẫn i) Đánh giá các phát biểu: -

-

(A) đúng. Các phản ứng SN1 cần các dung môi phân li proton với hằng số điện môi ε lớn. Vai trò của các dung môi như vậy (H2O, HCOOH, CH3OH, CH3COOH) là để làm bền hóa các cation và anion. (B) đúng. Với các phản ứng SN2, không thực sự cần dung môi protic, do chỉ có cation (ion bù trừ của nucleophile) cần được solvate hóa. Các dung môi tốt có thể kể đến là: acetone, các ether (diethyl ether, dioxane, tetrahydrofuran), acetonitrile, dimethylformamide, dimethyl sulfoxide, hexamethylphosphotriamide. (C) và (D) đúng, phản ứng vẫn có thể xảy ra nhưng với tốc độ rất chậm. (E) sai vì sự chọn lọc lập thể của các phản ứng thế của alkyl halide bậc hai có thể bị ảnh hưởng bởi dung môi.

ii) Do n-butyl bromide là alkyl halide bậc một nên phản ứng thế ưu tiên diễn ra theo kiểu SN2. Các dung môi protic (CH3OH, H2O) làm chậm tốc độ phản ứng, trong khi đó các dung môi aprotic phân cực (DMSO, DMF, CH3CN, [(CH3)2N]3P=O) thì phù hợp với phản ứng SN2. Các dung môi protic làm chậm tốc độ phản ứng SN2 bởi chúng solvate hóa nucleophile, làm giảm năng lượng và hoạt tính của nó. Các dung môi aprotic phân cực thì hòa tan được nhiều muối nhưng chúng solvate các cation thay vì anion có tính nucleophile. Kết quả là các anion không bị solvate hóa có thêm hoạt tính nucleophile và làm tăng tốc độ phản ứng SN2. iii) a) Dung môi với độ phân cực kém hơn (ethanol kém phân cực hơn nước) sẽ thuận lợi hơn.

b) Trong trường hợp này, trạng thái chuyển tiếp phân cực hơn, do đó dung môi phân cực hơn (chloroform) sẽ thuận lợi hơn.

173

4. Ảnh hưởng của nhóm rời đi i) Cho biết mối liên hệ giữa tính base và khả năng đóng vai trò như một nhóm rời đi: (A) Không có liên hệ gì cả. (B) Nhóm rời đi tốt là các base mạnh. (C) Nhóm rời đi tốt là các base yếu. ii) Sắp xếp các nhóm rời đi (được in đậm) theo mức độ dễ dàng thay thế trong các phản ứng thế nucleophile:

iii) Sắp xếp các nhóm rời đi dưới đây vào bảng mức độ dễ dàng tách chúng khỏi nguyên tử carbon của chất nền trong phản ứng thế nucleophile:

TsO− , Br − , H2O, OH− , I− , NH2− , Cl− , RO− , F− , NR3 Cực kì tốt Rất tốt Tốt Chấp nhận được Kém Rất kém iv) Ethyl halide nào sau đây tham gia phản ứng SN2 nhanh nhất?

v) Sắp xếp các hợp chất dưới đây theo trình tự hoạt tính trong các phản ứng SN2.

vi) Alkyl bromide nào sau đây có hoạt tính mạnh nhất trong phản ứng SN1?

174

vii) Carbocation nào sau đây có năng lượng cao nhất?

Hướng dẫn i) Các nhóm rời đi tốt là những base yếu của các acid liên hợp mạnh (C). ii) Một nhóm rời đi tốt nên là base liên hợp của acid mạnh.

iii) Cực kì tốt

TsO− , NR3

Rất tốt Tốt

I− ,Br − H2O

Chấp nhận được Cl− Kém F− Rất kém OH− , NH2− , RO− iv) Dãy hoạt tính của các ethyl halide trong phản ứng SN2: D > C > A > B. Trong phản ứng SN2, nucleophile tác kích vào nguyên tử carbon của chất nền từ phía ngược lại với nhóm rời đi. Theo khả năng dễ tách ra, các ion halide được sắp xếp như sau: I- > Br- > Cl- > F-. Dãy này tương ứng với sự tăng dần khả năng phân cực của ion halide (từ F- đến I-), nghĩa là xu hướng tách ra sẽ

175

tăng lên theo “tính mềm” của halogen. 1-Iodoethane (D) phản ứng nhanh nhất do ion iodide là nhóm rời đi tốt nhất trong dãy này. v) Hợp chất hoạt động nhất là methyl bromide do đây là alkyl halide bậc một với nhóm rời đi tốt (C > A > B > D). vi) Trong các phản ứng SN1, alkyl bromide C có hoạt tính mạnh nhất. Điều then chốt là phải biết phản ứng SN1 diễn ra như thế nào. Với phản ứng SN1, việc tách nhóm rời đi diễn ra trước khi tạo thành liên kết với nucleophile. Do đó, trung gian là carbocation. Hoạt tính của các alkyl halide R-Hal trong phản ứng SN1 tăng nếu điện tích dương của carbocation R+, tạo thành trong giai đoạn tốc định, được bền hóa (giải hỏa) bởi các hiệu ứng +I hoặc +M của các nhóm thế. Trong trường hợp này, carbocation bậc ba bền nhất. vii) Năng lượng càng cao thể hiện qua độ bền càng thấp. Độ bền phụ thuộc vào việc liệu có các nhóm nhường hoặc nhận electron liên kết với nguyên tử carbon dương điện không. Trong số các carbocation đã cho, carbocation bậc một (A) có năng lượng cao nhất – vì xung quanh không có nhóm nhường electron nào để giải tỏa điện tích.

176

5. Hóa lập thể i) Các phản ứng SN1 và SN2: (A) đều đặc trưng lập thể. (B) đều chọn lọc lập thể. (C) SN1 không đặc trưng lập thể, SN2 thì có. (D) SN1 đặc trưng lập thể, SN2 thì không. (E) đều không đặc trưng lập thể lẫn chọn lọc lập thể. ii) Biểu diễn cấu trúc các chất nền và sản phẩm thủy phân (R)-2-bromoctane trong một phản ứng SN1 lí tưởng. iii) Xác định sản phẩm của việc đun sôi (S)-3-chlorocyclopent-1-ene trong methanol. iv) Phát biểu nào sau đây về sự chuyển vị carbocation không chính xác: (A) Có thể diễn ra trong các phản ứng SN1. (B) Có thể diễn ra trong các phản ứng SN2. (C) Đôi khi bao gồm sự chuyển dịch nguyên tử hydrogen. (D) Diễn ra do độ bền cao hơn của các carbocation bậc hai. (E) Diễn ra nhanh hơn phản ứng giữa các phân tử. Hướng dẫn i) SN1 không đặc trưng lập thể, SN2 thì có (C). ii)

iii) Việc đun hồi lưu (S)-3-chlorocyclopent-1-ene trong methanol tạo ra điều kiện phù hợp với phản ứng SN1. Nucleophile tác kích vào carbocation phẳng dẫn đến sự tạo thành hỗn hợp racemic của các đồng phân ether.

iv) Ý sai là: Có thể diễn ra trong các phản ứng SN2 (B).

177

6. Sản phẩm phản ứng thế i) Thủy phân 2-bromo-3-methylbutane (I) và 3-bromo-2,2-dimethylbutane (II) trong điều kiện của các cơ chế phản ứng SN1 và SN2 lần lượt dẫn đến sự tạo thành các đồng phân A, B (từ I) và C, D (từ II). Xác định điều kiện của mỗi phản ứng, biểu diễn cấu tạo và đề xuất cơ chế tạo thành các hợp chất A, B và C, D.

ii) Đề xuất cơ chế phản ứng để giải thích các kết quả dưới đây:

iii) Giải thích hoạt tính tương đối của các hợp chất sau trong phản ứng thủy phân trong dung dịch ethanol.

iv) Xác định sản phẩm thủy phân 1-bromobut-2-ene trong các điều kiện phản ứng SN1 và SN2. v) Giải thích các dữ kiện thực nghiệm sau:

178

vi) Chất nào sau đây có hoạt tính kém nhất trong phản ứng SN2?

vii) Sắp xếp các alkyl bromide dưới đây theo trình tự tăng dần hoạt tính trong các phản ứng SN2.

(CH3 )3 CCH2 − Br

(CH3 )2 CH − Br

179

(CH3 )3 C − Br

CH3CH2 − Br

Hướng dẫn i) Tùy thuộc vào các điều kiện thủy phân mà 2-bromo-3-methylbutane (I) và 3-bromo-2,2-dimethylbutane (II) sẽ phản ứng theo cơ chế SN1 và SN2, lần lượt tạo thành các đồng phân A, B và C, D. Các phản ứng của nucleophile yếu (H2O) diễn ra theo cơ chế SN1, trong đó có sự chuyển vị của các carbocation. Còn khi sử dụng nucleophile mạnh (NaOH) thì diễn ra phản ứng SN2.

ii) Phản ứng SN2 của alkyl halide được tạo thuận lợi bởi nồng độ nucleophile mạnh trong dung môi aprotic phân cực. Dưới các điều kiện phản ứng SN2 (MeONa/DMF, nucleophile mạnh + dung môi aprotic), khi nguyên tử bromine bị thay thế bởi nhóm methoxy, cấu hình được bảo toàn với sự tạo thành chỉ hợp chất B.

180

Phản ứng SN1 của alkyl halide được tạo thuận lợi bởi các tính chất nucleophile yếu của methanol và khả năng ion hóa được tăng cường (cũng nhờ nhiệt độ cao) của nó trong vai trò dung môi. Dưới các điều kiện của phản ứng SN1 (MeOH/ 100 oC, nucleophile yếu + dung môi protic), nucleophile (methanol) tấn công vào nguyên tử carbon của carbocation phẳng với xác suất ở hai phía bằng nhau, dẫn đến sự tạo thành các lượng đẳng mol của các đối quang B và C.

iii) Hoạt tính tương đối cho thấy các bromide bậc ba A - C liên quan đến thực tế rằng theo điều kiện đã cho (thủy phân trong dung dịch ethanol), phản ứng diễn ra theo cơ chế SN1 qua carbocation phẳng. Không thể nhận được cation phẳng từ các hợp chất B và C (dạng hình học cực kì không thuận lợi, bị giới hạn bởi các vòng cứng nhắc), do đó, các cation tương ứng rất kém bền và phản ứng diễn ra chậm hơn nhiều. iv)

v) Sơ đồ tổng quát cho phản ứng SN1 có thể được biểu diễn như sau:

181

Nucleophile có thể tác kích bất kì dạng nào của vi hạt ion. Nếu nucleophile tác kích carbocation đã phân li hoàn toàn, sản phẩm sẽ được racemic hóa hoàn toàn. Mặt khác, nếu nucleophile tác kích hoặc cặp ion tiếp xúc, hoặc cặp ion đã tách ra-được solvate hóa, thì nhóm rời đi sẽ gây cản trở một phần sự tiếp xúc của nucleophile từ phía này của carbocation và do đó sẽ nhận được nhiều hơn sản phẩm nghịch đảo cấu hình (chú ý rằng nếu nucleophile tác kích vào phân tử chưa phân li thì sản phẩm sẽ có cấu hình nghịch đảo).

vi) Do sự án ngữ không gian nên hợp chất B có hoạt tính trong phản ứng SN2 kém nhất. vii) Yếu tố không gian đóng vai trò cực kì quan trọng.

182

7. Sản phẩm phản ứng thế i) Dự đoán những sản phẩm nào có thể được tạo thành khi dung môi phân neopentyl chloride trong HCOOH 80%. ii) Hãy dự đoán alkyl halide nào sau đây sẽ có hiệu suất cao nhất trong phản ứng thế với CH3CH2ONa tạo thành ether:

iii) Ether nào không thể được tạo thành bởi phản ứng Williamson?

Hướng dẫn i) Trong quá trình dung môi phân neopentyl chloride trong HCOOH 80%, phản ứng diễn ra theo cơ chế SN1. Carbocation tạo thành ở giai đoạn đầu tiên trải qua chuyển vị Wagner – Meerwein. Carbocation bậc ba mới phản ứng với nước tạo thành một alcohol bậc ba. Sự tách proton tạo thành các alkene.

183

ii) Halide A (1-bromopentane) có thể tạo thành ether (sản phẩm thế trong phản ứng Williamson) với hiệu suất tốt nhất. Các điều kiện phản ứng tương ứng với cơ chế SN2. Với phản ứng SN2, sự tác kích của nucleophile phải diễn ra theo hướng ngược lại của nhóm rời đi. Do đó, nguyên tử carbon không bị án ngữ không gian sẽ bị tác kích. Trong trường hợp có sự án ngữ không gian, ví dụ như bromide bậc ba C (1-bromo-2,2-dimethylpropane) phản ứng với CH3CH2ONa thì phản ứng tách chiếm ưu thế.

iii) Diphenyl ether (C) không thể được tạo thành bởi phản ứng Williamson, do các vinyl và aryl halide không trải qua phản ứng thế nucleophile. Có thể tổng hợp ester này dưới các điều kiện khác (ví dụ, khi có mặt muối đồng).

184

8. Phản ứng thế nucleophile Hãy đề xuất chuyển hóa (R)-butanol-2 thành các hợp chất sau: (A) (S)-2-fluorobutane (B) (R)-butanethiol-2 (C) (R, S)-sec-butylethyl ether Hướng dẫn

185

9. So sánh tốc độ phản ứng So sánh tốc độ phản ứng trong các dung môi khác nhau: a) Dung môi: C2H5OH (1) và DMSO (2).

b) Dung môi: H2O (1) và hỗn hợp H2O/acetone (2). c) Dung môi: CH3OH (1) và DMFA (2):

Chỉ ra kiểu cơ chế của các phản ứng trên. Hướng dẫn a) Cơ chế E2, tốc độ phản ứng trong DMSO (2) cao hơn trong C2H5OH (1). b) Cơ chế SN1, tốc độ phản ứng trong H2O (1) cao hơn trong hỗn hợp H2O/acetone (2). c) Cơ chế SN2, tốc độ phản ứng trong DMFA (2) cao hơn trong CH3OH (1).

186

10. So sánh tốc độ phản ứng 1) Sắp xếp các hợp chất sau theo trình tự giảm dần tốc độ dung môi phân trong formic acid:

2) Sắp xếp các hợp chất RI theo trình tự giảm dần hoạt tính trong phản ứng acetone →RI + KCl , với R = n-C3H7, sec-C4H9, iso-C4H9, CH3, sau: RCl + KI ⎯⎯⎯ PhCH2, CH2=CH, CH3COCH2, CH=CHCH2. 3) Hợp chất A có công thức phân tử C3H7ClO, dễ bị dung môi phân trong ethanol (0 oC, chu kì bán phản ứng 10 phút). Hợp chất B có cùng công thức phân tử nhưng dưới cùng điều kiện như trên thì phản ứng chậm hơn 10 lần so với hợp chất A. Hợp chất C, có công thức phân tử C3H7Cl, phản ứng với sodium phenoate trong ethyl alcohol ở 80 oC tạo thành một sản phẩm thế với tốc độ chậm hơn 10 lần so với ethyl chloride. Hãy đề xuất cấu tạo của các hợp chất A, B, C. 4) So sánh tốc độ dung môi phân trong formic acid của các cặp chất sau:

5) Sắp xếp các hợp chất RX theo trình tự giảm dần hoạt tính trong phản DMSO →RCN + KX ứng sau: RX + KCN ⎯⎯⎯ to

187

Hướng dẫn 1) Cơ chế SN1: (p-CH3O-C6H4)2CHCl ≫ (C6H5)3CCl > (C6H5)2CHCl > C6H5CH2Cl > (CH3)3CCl > CH3(C2H5)CHCl > CH3CH2CH2CH2Cl 2) Cơ chế SN2: CH3COCH2Cl > C6H5CH2Cl > CH2=CHCH2Cl > CH3Cl > CH3CH2CH2Cl > (CH3)2CHCH2Cl > CH3CH2CH(CH3)Cl ≫ CH2=CHCl 3) A: CH3CH(OH)CH2Cl; B: HOCH2CH2CH2Cl; C: (CH3)2CHCl 4)

5) [5] > [2] > [1] > [4] > [6] > [3] > [7]

188

11. Cơ chế phản ứng 1) Thủy phân allyl halide bậc ba A, có công thức phân tử C6H11Cl, và đồng phân allyl halide bậc một B của nó thì tạo thành hỗn hợp hai alcohol đồng phân C và D với tỉ lệ bằng nhau. Xác định công thức cấu tạo của A – D và viết phương trình phản ứng thủy phân, cho biết phản ứng thuộc loại cơ chế nào. 2) Xác định các sản phẩm (ít nhất ba) của phản ứng acetol phân (dung môi phân trong acetic acid) của cyclohexene-4-ol tosylate. 3) Phản ứng dưới đây khá thú vị khi xét trên phương diện cơ chế.

a) Dự đoán phản ứng đã xảy ra như thế nào bằng cách viết ra cơ chế từng giai đoạn. b) Dự đoán sản phẩm tạo thành khi thay chất đầu bằng 1,4dimethylbenzene.

189

Hướng dẫn 1) Phản ứng diễn ra theo cơ chế SN1:

190

2) Có thể tạo thành các sản phẩm I, II, III, IV, trong đó I và II là sản phẩm chính.

3) a) Trong giai đoạn đầu tiên, tert-butyl alcohol bị chuyển thành carbocation tương ứng. Tiểu phân này có thể mất 1 proton, tạo thành 2-methylpropene; hoặc lấy 1 hydride ion từ nhóm isopropyl của chất đầu. -

Hướng phản ứng thứ hai khả thi, bởi carbocation mới bền hơn tertbutylcarbocation do nó cũng là carbocation bậc ba, hơn nữa còn được bền hoá bởi hiệu ứng cộng hưởng của vòng thơm.

- Carbocation mới này phản ứng với 2-methylpropene giàu electron tạo thành một tiểu phân trung gian carbocation khác. Tiểu phân này có phản ứng thế electronphil nội phân tử với vòng thơm, tạo thành sản phẩm cuối.

191

b) Với 1,4-dimethylbenzene thì chỉ tạo thành 2,5-dimethyl-tert-butylbenzene qua một phản ứng thế electrophile thông thường do sự tách hydride ion từ nhóm methyl không ưu tiên xảy ra.

192

12. Cơ chế phản ứng Đưa ra lời giải thích ngắn gọn cho các quan sát sau đây: a) Trong những điều kiện giống nhau, phản ứng của NaSCH3 với 1-bromo2-methylbutane chậm hơn phản ứng với 1-bromobutane khá nhiều. b) Khi xử lý (S)-2-butanol tinh khiết quang học phản ứng với một base mạnh như LiNH2, rồi phục hồi, thì tính quang hoạt vẫn giữ nguyên. Nhưng nếu cho (S)-2-butanol phản ứng với nước ấm, có một lượng nhỏ sulfuric acid, thì sau khi phục hồi, alcohol bị mất tính quang hoạt. c) Phản ứng của cyclobutene với bromine (Br2, lạnh, trong bóng tối) tạo thành một hỗn hợp racemic. Trong khi đó, phản ứng với hydrogen “nặng” khi có xúc tác platinum (D2/Pt) lại tạo thành một hợp chất meso. d) Đun hồi lưu (R)-2-bromobutane với dung dịch NaOH đặc trong ethanol, thu được (S)-2-butanol. e) Đun hồi lưu (R)-2-bromobutane với dung dịch NaOH loãng trong ethanol thu được hỗn hợp racemic của 2-butanol. Tốc độ tạo thành alcohol thay đổi thế nào khi: i. tăng gấp đôi nồng độ alkyl bromide; và ii. tăng gấp đôi nồng độ NaOH? f) Trong các điều kiện giống nhau, phản ứng của các đồng phân dia (xuyên lập thể phân) A, B tạo thành các sản phẩm khác nhau rõ rệt. Gợi ý: Hãy chú ý đến khía cạnh lập thể không gian ba chiều.

193

Hướng dẫn a) Cả hai phản ứng đều thuộc loại SN2, xảy ra ở nguyên tử carbon bậc một. Phản ứng SN2 chịu ảnh hưởng chính bởi các tương tác không gian. 1-bromo2-methylbutane phân nhánh ở vị trí , gây ra sự cản trở không gian khi tác nhân nucleophile thiolate tấn công. Do vậy phản ứng của nó chậm hơn 1bromobutane. b) Phản ứng đầu tiên chỉ có sự deproton hoá/proton hoá và chỉ ảnh hưởng tới liên kết O-H chứ không làm thay đổi cấu hình của tâm chiral. Do vậy nó không bị mất tính quang hoạt. Phản ứng thứ hai có sự tham gia của liên kết C-O của tâm chiral và có sự hình thành một tiểu phân carbocation với cấu trúc phẳng. Khi cộng hợp nước vào carbocation phẳng này thì phản ứng có thể xảy ra ở cả hai phía, dẫn tới sự hình thành một hỗn hợp racemic và làm mất tính quang hoạt.

c) Phản ứng cộng bromine là quá trình anti (trái phía), dẫn tới sự tạo thành một sản phẩm trans. Sự tấn công của ion Br+ vào hai mặt (trên và dưới) của cyclobutene đều tạo ra cùng một tiểu phân bromonium ion. Tiếp đó, bromide ion tấn công, mở vòng tiểu phân này, tạo thành một hỗn hợp sản phẩm (R,R)and (S,S)- với số mol bằng nhau (racemate), hỗn hợp này không có tính quang hoạt. Phản ứng hydrogen hoá lại là quá trình syn (cùng phía), dẫn tới sự tạo thành một sản phẩm cis. Khả năng cộng hợp vào hai phía của cyclobutene là tương đương nhau, nhưng lần này sản phẩm tạo thành lại chỉ là một hợp chất meso. Sản phẩm này cũng không có tính quang hoạt.

194

d) Tác nhân nucleophile (OH-) có nồng độ cao thì phản ứng ưu tiên xảy ra theo cơ chế SN2, dẫn tới sự nghịch đảo cấu hình 100 % ở vị trí xảy ra sự thế. e) Nồng độ nucleophile thấp thì phản ứng ưu tiên xảy ra theo cơ chế SN1, đi qua trạng thái carbocation. Tiểu phân trung gian phẳng này có thể “bắt” dung môi hoặc tác nhân nucleophile ở cả hai phía, dẫn tới sự racemic hoá. Trong các điều kiện đã cho, tốc độ phản ứng SN1 chỉ phụ thuộc vào nồng độ alkyl halide, do vậy khi tăng gấp đôi nồng độ alkyl halide thì tốc độ tạo ra alcohol tăng gấp đôi Còn tăng gấp đôi nồng độ nucleophile (OH-) không làm ảnh hưởng đến tốc độ phản ứng. f) Phản ứng (1) xảy ra theo cơ chế SN2, trong đó tác nhân nucleophile (ion CH3O-) thay thế cho bromide anion, dẫn tới sự nghịch đảo cấu hình. Phản ứng tách không thể xảy ra bởi không tồn tại cấu dạng nào có nguyên tử Br và nguyên tử hydrogen của carbon liền kề đều ở vị trí liên kết trục (sự sắp xếp anti-periplanar). Phản ứng (2) xảy ra theo cơ chế tách E2. Phản ứng này dễ xảy ra bởi sự “lật vòng” tạo thành cấu dạng với sự sắp xếp anti-periplanar của hydrogen và bromine.

195

13. Hóa lập thể của phản ứng thế A. Phản ứng của (R)-2-hydroxysuccinic acid với phosphorus trichloride tạo thành sản phẩm B.

1) Xác định cấu trúc (R)-2-hydroxysuccinic acid. 2) Xác định cấu trúc trung gian A và sản phẩm B, chỉ rõ hóa lập thể. 3) Đề xuất cấu trúc trạng thái chuyển tiếp trong các chuyển hóa của A, B. B. Phản ứng của (1S, 2S)-trans-p-bromobenzenesulfonate của iodocyclohexyl (W) với acetic acid tạo thành trans-acetate của 2iodocyclohexyl (T). 1) Viết cấu dạng ghế của W(e,e) và W(a,a). Với e là equatorial (biên), còn a là axial (trục). Phản ứng diễn ra theo sơ đồ dưới đây:

2) Xác định các trung gian X+, Y+ và T trong giản đồ trên và viết cấu dạng của chúng. 3) Xác định cấu hình R/S của các nguyên tử carbon thủ tính trong T.

196

Hướng dẫn A.1)

2)

3)

B.1)

197

2)

198

14. Xác định hợp chất hữu cơ Hợp chất A (C5H8O, đối quang S tinh khiết), có thể được chuyển thành hợp chất B (C5H7Br, đối quang R tinh khiết) qua một quá trình hai giai đoạn: i) CH3SO2Cl, triethylamine ii) LiBr Khi phản ứng với khi hydrogen, có xúc tác kim loại, B chuyển thành hai phân tử phi thủ tính C, D (đều có công thức phân tử C5H9Br). Chuyển B thành hợp chất Grignard tương ứng, rồi thuỷ phân với nước, thu được hợp chất E (C5H8, phi thủ tính). Cho E phản ứng với dung dịch acid của KMnO4, tạo thành F (C5H8O3). Phổ hồng ngoại của F cho thấy có sự tồn tại của 2 nhóm carbonyl khác nhau và 1 nhóm hydroxy. Xác định cấu trúc hoá lập thể của các hợp chất A-F. Hướng dẫn

199

15. Chuyển hóa hữu cơ 1) Tiến hành chuỗi chuyển hóa như sau với (R)-1-phenylpropanol-2 chứa đồng vị đánh dấu 18O:

Xác định cấu trúc các chất A, B, chỉ rõ cấu hình tâm thủ tính. Chất nào trong các sản phẩm cuối có chứa đồng vị đánh dấu? 2) So sánh góc quay của mặt phẳng ánh sáng phân cực đối với ether B, được tạo thành theo hai cách khác nhau từ alcohol quang hoạt:

3) Hoàn thành chuỗi chuyển hóa sau và chỉ ra cấu hình tuyệt đối (R/S) của các hợp chất nhận được ở mỗi giai đoạn:

4) Hoàn thành chuỗi chuyển hóa sau và chỉ ra cấu hình tuyệt đối (R/S) của các hợp chất nhận được ở mỗi giai đoạn:

200

Hướng dẫn 1)

2)

3)

Cơ chế phản ứng thế với KOH/H2O-EtOH (bảo toàn cấu hình):

4)

201

16. Chuyển hóa hữu cơ 1) Hoàn thành chuỗi chuyển hóa sau:

2) Hoàn thành chuỗi chuyển hóa sau:

3) Hoàn thành chuỗi chuyển hóa dưới đây:

Hướng dẫn 1)

202

2)

Hiệu suất của D cao hơn đáng kể so với G do cấu dạng I, trước khi diễn ra phản ứng tách E2, thuận lợi về mặt năng lượng hơn cấu dạng II – với các nhóm phenyl cồng kềnh ở gần nhau.

3)

203

17. Ảnh hưởng của nhóm kề 1) Hoàn thành chuỗi chuyển hóa sau. Đề xuất cơ chế khả dĩ cho bước cuối cùng.

2) Trình bày cơ chế đồng phân hóa các p-haloamine:

Tại sao cân bằng chiếm ưu thế bởi halide chứa nguyên tử halogen ở carbon bậc hai? Tại sao phản ứng thủy phân các β-chloroamine R2NCH2CH(Cl)CH3 trong các môi trường kiềm và acid lại tạo thành các β-aminoalcohol đồng phân?

3) Tốc độ dung môi phân trong acetic acid (acetol phân) khi có mặt sodium acetate của các đồng phân hình học của 2-acetoxycyclohexyl tosylate có sự khác biệt rõ rệt: đồng phân trans phản ứng nhanh hơn 670 lần so với đồng phân cis. Ngoài ra, trong cả hai trường hợp, diacetate tạo thành đều có cấu hình trans.

204

Phản ứng acetol phân dưới các điều kiện này của trans-2acetoxycyclohexyl tosylate quang hoạt tạo thành racemic của transdiacetate. Hãy giải thích: (1) Sự khác biệt rõ ràng giữa tốc độ acetol phân của các đồng phân cis và trans tosylate. (2) Kết quả hóa lập thể quan sát được từ phản ứng acetol phân của các đồng phân này. (3) Sự tạo thành trans-diacetate trong phản ứng acetol phân của trans2-acetoxycyclohexyl tosylate quang hoạt. 4) Giải thích sự khác nhau về kết quả hóa lập thể trong phản ứng của methyl ester của (S)-2-bromopropionic acid với sodium methoxide và muối sodium của (S)-2-bromopropionic acid với methanol.

Việc thêm muối bạc vào sẽ ảnh hưởng gì đến tốc độ phản ứng thứ hai? Điều này có làm thay đổi kết quả hóa lập thể không?

205

Hướng dẫn 1)

2) Cơ chế phản ứng đồng phân hóa:

Trong môi trường acid mạnh, nguyên tử nitrogen bị proton hóa hoàn toàn và cặp electron chưa liên kết của nó không tham gia vào quá trình thủy phân dẫn xuất halogen. Do đó, phản ứng diễn ra như một phản ứng thế bình thường, nghĩa là không có sự tham gia của nhóm kề. Còn nếu amine không bị chuyển thành dạng muối thì giai đoạn đầu tiên của quá trình thủy phân là phản ứng thế SN2 nội phân tử, tạo thành ion immonium A. Khi có mặt base mạnh, ion này tiếp tục trải qua phản ứng mở vòng theo cơ chế SN2 ở nguyên tử carbon bậc một hoạt tính mạnh hơn, tạo thành sản phẩm chuyển vị.

206

3) Sự khác biệt rõ rệt về tốc độ acetol phân được giải thích bởi sự tham gia của nguyên tử oxygen trong hợp phần carbonyl (nhóm carboxyl) trong trường hợp đồng phân trans, còn trong trường hợp đồng phân cis thì không có sự tham gia của nhóm kề.

4) Kết quả hóa lập thể của phản ứng thứ hai sẽ không bị ảnh hưởng khi thêm muối bạc, nhưng tốc độ phản ứng thì sẽ tăng lên, bởi ion bạc sẽ tạo phối trí với bromine, giúp tăng tốc độ tách loại ion bromide này.

207

18. Cơ chế phản ứng Đề xuất cơ chế cho các phản ứng sau:

Hướng dẫn a)

b)

208

c)

d)

e)

f)

209

19. Cơ chế phản ứng Đề xuất cơ chế các phản ứng sau:

210

Hướng dẫn a)

b)

c)

d)

211

19A. Cơ chế phản ứng Từ khung bixiclo[4.1.0]heptane có thể mở rộng vòng để tạo thành hệ vòng 7 cạnh. a) Cho biết cấu trúc của A, B trong hai phản ứng dưới đây

b) Giải thích sự tạo thành một trong hai sản phẩm dựa trên cấu dạng của chất đầu. Hướng dẫn Cấu trúc các sản phẩm

Do vòng ba cạnh phẳng nên nó buộc các nguyên tử carbon số 1, 2, 3 và 4 cũng phải phẳng (tương tự vòng cyclohexane). Như vậy cấu dạng chính xác phải là dạng bán ghế thay vì dạng ghế thông thường,

Carbocation không cổ điển Giai đoạn nước cộng vào carbocation không cổ điển có thể được biểu diễn chính xác hơn như sau:

212

20. Cấu dạng và phản ứng tách Sự chuyển vị của các phân tử hoạt tính là loại phản ứng hữu cơ cực kì quan trọng (đặc biệt trong các phản ứng xúc tác bởi enzyme). Việc nghiên cứu thường tập trung vào thành phần hoạt tính của phân tử, kiểu và tốc độ phản ứng. 1) So sánh các phân tử benzene và cyclohexane (hình 1, hình 2): chỉ ra trạng thái lai hóa của mỗi nguyên tử carbon và góc gần đúng giữa hai liên kết C-C cạnh nhau trong mỗi phân tử. Các phân tử này có phẳng hay không.

Cấu dạng (sự sắp xếp không gian) bền nhất của cyclohexane là cấu dạng ghế. Có hai cấu dạng như vậy và chúng tồn tại trong một cân bằng (hình 3). Ở cấu dạng trái, các nguyên tử HA và HD ở vị trí trục, còn các nguyên tử HB và HC ở vị trí biên. Sự nghịch đảo vòng diễn ra làm vị trí của các nguyên tử hydrogen thay đổi. Các cyclohexane có nhóm thế duy trì cấu dạng mà các nhóm thế lớn ở vị trí biên, bởi cấu dạng này bền về mặt năng lượng hơn. Trong tất cả các câu hỏi tiếp theo, hãy vẽ tất cả các cấu dạng theo cách như trong hình 3. 2) Vẽ các cấu dạng ghế cho mỗi hợp chất dưới đây. Chỉ ra đâu là cấu dạng bền hơn.

3) Vẽ cấu dạng ghế bền nhất của cis-decalin và trans-decalin (bao gồm cả các nguyên tử hydrogen được dẫn ra) và chỉ ra hợp chất nào, cis-decalin hay trans-decalin, bền hơn?

213

4) Xác định sản phẩm của các phản ứng tách E2 sau đây. Đề xuất cơ chế của các phản ứng (sử dụng các cấu dạng ghế). Một trong các phản ứng không diễn ra, hãy cho biết đó là phản ứng nào và giải thích tại sao.

Hướng dẫn 1) Benzene: các nguyên tử carbon đều lai hóa sp2, góc giữa các liên kết C-C là 120o, phân tử phẳng. Cyclohexane: các nguyên tử carbon đều lai hóa sp3, góc giữa các liên kết CC là 109.5o, phân tử không phẳng. 2) a) Hai cấu dạng như nhau, không thể so sánh tính bền:

b) Cấu dạng bên phải bền hơn:

c) Cấu dạng bên trái bền hơn: 214

3)

Trans-decalin bền hơn bởi tất cả các nhóm thế alkyl đều ở vị trí biên, trong khi đó cis-decalin có một nhóm thế alkyl ở vị trí trục và một nhóm biên. 4) Để phản ứng tách E2 diễn ra, cả nguyên tử hydrogen bị base tác kích lẫn nhóm rời đi phải ở trên các nguyên tử carbon cạnh nhau và phải ở vị trí trục. Phản ứng (c) không diễn ra bởi nguyên tử bromine và nhóm tert-butyl cồng kềnh phải ở vị trí trục, rất không thuận lợi về mặt năng lượng.

215

21. Tổng hợp thyroxine Thyroxine (K) là một hormone tiết ra từ tuyến giáp.

Đi từ tyrosine (А), hợp chất K được tổng hợp theo sơ đồ sau:

Phản ứng của tyrosine (А) với nitric acid được tiến hành trong các điều kiện như vậy sẽ diễn ra sự thế hai lần. Hợp chất B tạo thành ở dạng ion lưỡng cực. B phản ứng với acetic anhydride theo tỉ lệ mol 1:1 tạo thành C. Hợp chất C có kết quả dương tính với thuốc thử FeCl3. Sự khử F thành G diễn ra với lượng dư hydrogen. Các chuyển hóa G → H và I → J cũng sử dụng tác nhân dư. Hợp chất J phản ứng với I2 theo tỉ lệ mol 1:2. 1) Xác định cấu hình tuyệt đối (R/S) cho các tâm lập thể của hợp chất A theo quy tắc Cahn-Ingold-Prelog. 2) Hoàn thành tất cả các phản ứng trong sơ đồ trên, sử dụng cấu trúc ba chiều để biểu diễn các chất B-K. Đưa ra kí hiệu dạng cơ chế (SN, E, …) cho các chuyển hóa sau: A → B, B → C, C → D, E → F. 3) Đề xuất cơ chế chi tiết cho các chuyển hóa: a) A → B (giai đoạn nitro hóa đầu tiên); b) B → C, c) C → D. 4) Xác định cấu hình tuyệt đối (R/S) cho các tâm lập thể của hợp chất K theo quy tắc Cahn-Ingold-Prelog.

216

Hướng dẫn 1) Cấu hình S. 2)

Cơ chế phản ứng nitro hóa là SEAr.

Cơ chế phản ứng là AАс2.

Cơ chế phản ứng là ААс2.

217

Cơ chế phản ứng là SNAr.

Cơ chế phản ứng là SEAr.

218

3) a)

Phản ứng thế thứ hai cũng tương tự. b)

219

c)

d) Hợp chất K có cấu hình S.

220

22. Tổng hợp amine 1) Đề xuất phương pháp tổng hợp tert-butylamine, đi từ tert-butanol, cùng các tác nhân cần thiết khác. 2) Từ benzene và các tác nhân cần thiết khác, hãy tổng hợp 1,3dibenzoylbenzene. 3) Từ p-toluidine (4-methylaniline) và các tác nhân cần thiết khác, hãy tổng hợp 2-hydroxy-4', 5-dimethyl-2', 3-dinitroazobenzene (A).

4) Hãy điều chế 2-hydroxy-3-fluorobenzaldehyde từ o-nitroaniline và các tác nhân cần thiết khác. 5) Từ cyclohexanone và các tác nhân cần thiết khác, hãy tổng hợp amine bậc ba A có cấu trúc như sau:

221

Hướng dẫn 1)

2)

3)

222

4)

5)

223

23. Xác định amine Amine C7H13N (A) tạo thành phức chất với B(CH3)3 mạnh hơn phức của triethylamine và không tạo thành sản phẩm bền trong phản ứng với benzenesulfonyl chloride trong kiềm. Khi được đưa vào chuỗi thoái phân sau đây thì tạo thành trivinylmethane.

Xác định cấu trúc các hợp chất A, B, C. Hướng dẫn Theo đề bài đã cho, amine C7H13N (A), phải là một amine bậc ba với cấu trúc cứng nhắc, được cố định bởi hệ vòng bicyclic, trong đó cặp electron chưa liên kết (trong orbital lai hóa sp3) không bị án ngữ bởi bộ khung carbon. A phải là amine bậc ba bởi nó không tạo thành sản phẩm bền với C6H5SO2Cl trong dung dịch kiềm (sản phẩm cộng này bị phân cắt bởi dung dịch NaOH, hoặc thậm chí là chỉ bởi nước). Cấu trúc của A có thể được suy luận từ ba giai đoạn liên tiếp của phản ứng tách muối ammonium hydroxide bậc bốn theo Hofmann:

224

24. Xác định amine Thiết lập cấu trúc của aminoalcohol có công thức C8H15NO (A), chứa một nhóm hydroxyl bậc hai, dựa vào các thông tin từ chuyển hóa sau:

Biết muối D chứa một cation thơm, bền và không quang hoạt. Hướng dẫn Sản phẩm cuối của chuỗi thoái phân có chứa cation thơm, bền (C7H7+) – nhiều khả năng là cation tropylium, do đó D là tropylidene (cycloheptatriene1,3,5), còn C là cycloheptadiene với một nhóm dimethylamino, bị tách ra trong thoái phân Hofmann.

225

Chương 4

Hợp chất carbonyl và dẫn xuất 1. Phản ứng của các aldehyde và ketone Cho sơ đồ phản ứng sau:

a) Xác định cấu trúc của A, B và các tác nhân c, d. b) Phản ứng tạo thành A từ benzaldehyde thuộc loại nào? Trình bày cơ chế phản ứng. Các alcohol cũng phản ứng với aldehyde và ketone. Trước tiên, tạo thành các hemiacetal, rồi đến acetal: +

+

H H Aldehyde / ketone + alcohol ⎯⎯ →hemiacetal ⎯⎯ →acetal

Phản ứng tạo thành một acetal diễn ra qua nhiều giai đoạn:

c) Xác định cấu trúc của K-N. d) Làm thế nào để cân bằng chuyển dịch theo chiều tạo thành acetal?

226

Các alcohol với nhiều hơn một nhóm -OH như glycerol (propane-1,2,3-triol) cũng phản ứng với ketone tạo thành acetal. e) Viết phương trình phản ứng của acetone và glycerol. Xét các phản ứng sau của benzaldehyde:

f) Xác định cáu trúc O-T. Phản ứng cuối ở ý e khi benzaldehyde tạo thành S và T là một ví dụ của phản ứng Cannizzaro. Trong phản ứng này, carbon tự oxi hóa-khử (dị phân). Các aldehyde không có α hydrogen thì có thể phản ứng theo cách này. Giai đoạn đầu tiên của phản ứng Cannizzaro tạo thành một trung gian tứ diện. g) Trình bày cơ chế phản ứng Cannizzaro, lấy ví dụ phản ứng của benzaldehyde. h) Giải thích tại sao các hợp chất có α hydrogen thì không xảy ra phản ứng Cannizzaro.

227

Hướng dẫn a)

b) Phản ứng thế nucleophile.

c)

d) Có thể dùng thiết bị tách nước để chuyển cân bằng theo chiều tạo thành sản phẩm. e)

228

f)

g)

h) Các aldehyde với hydrogen ở vị trí alpha sẽ tạo thành enol khi xử lí với base. Do đó không thể diễn ra phản ứng Cannizzaro.

229

2. Các hợp chất carbonyl ở dạng chất phản ứng và sản phẩm Các hợp chất carbonyl là những nguồn giá trị đối với tổng hợp hữu cơ bởi chúng phản ứng được với nhiều hợp chất khác nhau, thường thì có độ chọn lọc. Trong sơ đồ phản ứng dưới đây, một số phản ứng đi từ butyl alcohol (1) đã được mô tả:

a) Vẽ các cấu trúc A đến G. b) Đưa ra một phương án khả thi để i) tổng hợp trực tiếp B từ butyl alcohol. Hợp chất F là một epoxide. c) Xác định tác nhân ii) cần để tổng hợp diol 2 và tác nhân iii) để chuyển hóa diol 2 thành A. Gọi tên phản ứng của diol 2 thành A. (Gợi ý: Hợp chất iii) có chứa một nguyên tử halogen ở số oxi hóa cao nhất). d) Dự đoán sản phẩm từ phản ứng của hợp chất C với MeLi. Vẽ cấu trúc sản phẩm. e) Tại sao không thể tạo ra hợp chất D bằng cách cho B phản ứng trực tiếp với MeLi? Cuối cùng hợp chất D được chuyển thành 5. f) Đề xuất các điều kiện phản ứng iv) cho phản ứng của D thành 5.

230

g) Trình bày cơ chế phản ứng này và cho biết chất trung gian và sản phẩm thuộc nhóm hợp chất nào? Hướng dẫn a)

b) i: oxi hóa với K2Cr2O7, H2SO4 loãng (phản ứng oxi hóa Jones), hoặc với CrO3, H2SO4 loãng, acetone hoặc với KMnO4 c) ii: Acid (H3O+) hoặc base (OH-) để mở vòng epoxide. iii: Phân cắt periodate với NaIO4. d)

e) MeLi chỉ có phản ứng dữ đội đi kèm với sự deproton hóa acid. f) iv: Ethane diol, H+ g)

231

3. Nhận biết hợp chất hữu cơ Các hợp chất sau chứa trong 5 bình mất nhãn. Một loạt các thí nghiệm xác định nhóm chức đã được thực hiện với các chất có trong 5 bình. Dựa vào các kết quả thực nghiệm, hãy xác định các chất sau có trong bình nào.

i) Lấy 5 ống nghiệm nhỏ, cho vào mỗi ống 1 mL acetone. Thêm khoảng 10 – 20 mg mỗi chất cần xác định vào mỗi ống nghiệm, tiếp đó nhỏ một giọt thuốc thử chromic-sulfuric acid rồi lắc nhẹ để trộn lẫn các chất với nhau. Sau vài phút, mẫu chứa A và C phản ứng với dung dịch dichromate tạo thành dung dịch màu xanh ngọc lục bảo và một kết quả có thể nhìn thấy. Để xác nhận màu xanh ngọc lục bảo của kết tủa, phần váng nổi lên được gạn cẩn thận rồi cho vào ống nghiệm. Kết tủa được rửa hai lần cho đến khi xuất hiện màu sắc rõ ràng. ii) Lặp lại thí nghiệm (i), nhưng thay thuốc thử chromic-sulfuric acid bằng một giọt dung dịch KMnO4 0.2 %, thì lại xảy ra sự chuyển màu và chỉ quan sát được có kết tủa tạo thành trong các mẫu chứa A, C. iii) Khi thêm dung dịch sodium hydroxide loãng vào các ống nghiệm rồi lắc nhẹ để trộn lẫn các chất thì chỉ có mẫu chứa B tan. B cũng là hợp chất duy nhất có phản ứng dương tính với giấy quỳ. iv) Khi thêm dung dịch sodium hypoiodite (được điều chế bằng cách hoà tan iodine vào dung dịch sodium hydroxide) vào các ống nghiệm thì chỉ mẫu chứa A và E tạo ra kết tủa vàng tươi. v) Các hợp chất C, D, E phản ứng với thuốc thử 2,4-dinitrophenyl hydrazine (2,4-DNPH) tạo ra kết tủa đỏ cam.

232

Hướng dẫn i) Phản ứng oxi hóa bởi chromate để xác định những nhóm chức dễ bị oxi hóa như alcohol, aldehyde. Do vậy, chất 4, 5 phải là A, C (mặc dù chưa biết chính xác từng chất.) ii) Phản ứng oxi hóa bởi permanganate cũng để xác định những nhóm chức dễ bị oxi hóa. Thí nghiệm này chỉ đơn thuần là củng cố lại những nhận định từ i. iii) Tan trong base và dương tính với giấy quỳ là tính chất của nhóm acid. Do vậy B phải là chất có nhóm chức carboxylic acid – 2. iv) Phản ứng với iodoform là của methyl ketone và methyl alcohol bậc II (chất này bị thuốc thử oxi hóa thành methyl ketone), do vậy A, E phải là 3, 4 . Kết hợp với dữ kiện ở i, ii ta xác định được các cặp chất: C-5; A-4; E-3. Chỉ còn lại D, nó chỉ có thể là 1. v) Phản ứng với 2,4-dinitrophenylhydrazine để xác định nhóm chức aldehyde và ketone. Chỉ có 1, 3, 5 có phản ứng này. Kết quả này xác nhận những kết luận trên là hợp lí.

233

3. Tổng hợp hữu cơ Từ methyl ethyl ketone và propionaldehyde, cùng các tác nhân cần thiết khác, hãy điều chế: -

2,3-dimethylpent-2-enal [A] 2,3-dimethylpentanal [B] 2,3-dimethylpent-2-en-1-ol [C]

Hướng dẫn Trước tiên, để điều chế aldehyde không no α, β A thì cần thực hiện phản ứng ngưng tụ aldol-croton trực tiếp giữa các hợp chất carbonyl khác nhau: propionaldehyde (hợp phần methylene) và methyl ethyl ketone (hợp phần carbonyl). Sau đó, khử hóa chọn lọc liên kết đôi (để điều chế B) hoặc nhóm carbonyl (điều chế C).

Liên kết đôi carbon-carbon trong aldehyde không no α, β A cũng có thể được khử chọn lọc bởi phản ứng hydrogen hóa xúc tác ở 20 oC và áp suất thông thường trên xúc tác palladium, ruthenium, hoặc platinum (sản phẩm là aldehyde B). Sự khử hóa nhóm carbonyl của các α, β-enone và enal đạt được khi sử dụng tác nhân khử diisobutylaluminium hydride (DIBAL-H) trong toluene dưới các điều kiện rất êm dịu (sản phẩm là alcohol C).

234

3A. Điều chế nhóm chức ester Nhóm chức ester khá phổ biến trong các hợp chất hữu cơ, và các nhà hoá học cũng đã tìm ra nhiều phương pháp để tổng hợp nhóm chức quan trọng này. Nhiều trong số đó có cơ chế tương tự nhau và đều liên quan đến sự thế nucleophile của acyl.

Ngoài ra cũng có một số hướng tổng hợp khác. Dưới đây là hai phản ứng có cơ chế tươg tự nhau. Hãy trình bày cơ chế của hai phản ứng này:

235

Hướng dẫn Cả hai phản ứng đều có giai đoạn chuyển carboxylic acid thành muối carboxylat (có tính nucleophile mạnh hơn), sau đó chất này tham gia phản ứng thế SN2, tạo thành ester. Trong trường hợp đầu tiên, iodide anion là nhóm rời đi. Còn ở trường hợp sau thì carboxylate (tạo thành khi proton chuyển qua diazomethane) thay thế cho N2.

236

4. Các enol và enolate trong hóa hữu cơ Sơ đồ 1 biểu diễn một hiện tượng mà một mặt là cơ sơ của các phản ứng hữu cơ quan trọng, nhưng mặt khác lại thường gây ra những phản ứng phụ không mong muốn. Dưới đây là một số câu hỏi liên quan đến hiện tượng này.

Sơ đồ 1 1. Xử lí isopropyl phenyl ketone với LDA (Lithium diisopropylamide), sau đó alkyl hóa với dimethyl sulphate (xem sơ đồ 2). Kết quả của phản ứng này là hai sản phẩm A và B được tạo thành. Tỉ lệ của các sản phẩm có thể thay đổi bằng cách điều chỉnh điều kiện phản ứng. Các dung môi aprotic, phân cực (ví dụ như HMPA, hexamethylphosphoramide) có thể làm tăng tỉ lệ sản phẩm B. Dữ kiện 1H NMR của hợp chất A: 5H, mũi đa, 7 ppm; 3H, mũi đơn, 3.3 ppm; 3H, mũi đơn, 1.8 ppm; 3H, mũi đơn, 1.7 ppm. Dữ kiện 1H NMR của hợp chất B: 5H, mũi đa, 7 ppm và 9H, mũi đơn, 1.2 ppm.

Sơ đồ 2 2. Ketone không no α, được biểu diễn trong sơ đồ 3, trải qua phản ứng cộng 1,2 hoặc 1,4 (cộng Michael) tùy vào nucleophile được sử dụng. Phản ứng với tác nhân Grignard tạo thành hợp chất C, trong khi đó phản ứng với thiophenol tạo thành hợp chất D.

Sơ đồ 3

237

3. Xử lí cyclohexanone và methyl vinyl ketone với base thì xảy ra phản ứng vòng hóa Robinson (sơ đồ 4). Đấy là một quá trình gồm ba giai đoạn. Trước tiên là phản ứng cộng Michael đã được đề cập ở trên, sau đó trung gian E tạo thành trải qua phản ứng aldol hóa nội phân tử, tạo thành hợp chất F, cuối cùng chất này trải qua phản ứng tách tạo thành sản phẩm vòng hóa Robinson (G) với công thức C10H14O. Các dữ kiện phổ hồng ngoại và 1H NMR quan trọng nhất của hợp chất G: Quan sát được trong phổ hồng ngoại một dải mạnh ở 1705 cm-1; trong phổ 1H NMR, quan sát được một số tín hiệu đặc trưng của các proton liên kết với các nguyên tử carbon lai hóa sp3 và một tín hiệu đặc trưng của proton liên kết với nguyên tử carbon lai hóa sp2. (Chú ý: Việc giải thích dữ liệu phổ đóng vai trò củng cố lập luận và không đòi hỏi phải có trong lời giải cho bài này.)

a) Cho biết tên gọi hiện tượng trong Sơ đồ 1. b) Xác định cấu trúc A, B. c) Giải thích hiệu ứng dung môi đến sự phân bố sản phẩm alkyl hóa được mô tả ở ý 1. d) Trong câu: “LDA là base Brönsted mạnh/yếu và là nucleophile mạnh/yếu” - hãy gạch bỏ hai từ tạo ra một phát biểu đúng. Giải thích ngắn gọn. e) Sử dụng thuyết acid/base cứng/mềm (HASB) để giải thích cấu trúc của các sản phẩm C, D được tạo thành. f) Xác định cấu trúc của các trung gian C, D và sản phẩm vòng hóa G. Hướng dẫn a) Hiện tượng hỗ biến (tautomer hóa) keto-enol. b)

238

c) Trong trường hợp này, phản ứng O-alkyl hóa chiếm ưu thế, khi enolate bị phân li (nhờ động lực từ HMPA, một dung môi aprotic phân cực), nó có thể tạo phức với các lithium ion. Còn tert-butyl alcohol có thể tạo thành các liên kết hydrogen với nguyên tử oxygen trong enolate, làm cản trở phản ứng Oalkyl hóa và ưu tiên phản ứng C-alkyl hóa. d) LDA là base mạnh, bởi nó là base liên hợp của acid yếu (diisopropylamine), nhưng lại là tác nhân nucleophile yếu vì sự án ngữ không gian gây ra bởi các nhóm thế isopropyl. “LDA là base Brönsted mạnh/yếu và là nucleophile mạnh/yếu”.

e) Các tác nhân Grignard là các nucleophile cứng, theo thuyết HSAB thì nó ưu tiên tấn công các tâm electrophile cứng, như là nguyên tử carbonyl carbon. Còn thiophneol là nucleophile mềm, ưu tiên phản ứng với các tâm electrophile như là nguyên tử vinyl carbon (beta).

f)

239

240

4A. Phản ứng ngưng tụ Cho 1 mol semicarbazide (A) phản ứng với 1 mol cyclohexanone (B) và 1 mol furtural (C) trong ethanol có một lượng nhỏ acid (dạng vết) thì thu được một hỗn hợp semicarbazone (D, E). Nếu kết thúc phản ứng sau 5 phút thì hỗn hợp chủ yếu chứa semicarbazone D. Tuy nhiên, nếu thực hiện phản ứng qua đêm, thì trong sản phẩm gần như chỉ có semicarbazone E. Sử dụng giản đồ năng lượng để giải thích cho các kết quả này.

Hướng dẫn Nhóm carbonyl trong cyclohexanone hoạt động hoá học (tính electrophile) hơn trong furtural do furtural được bền hoá bởi hiệu ứng liên hợp với vòng thơm. Do đó, nếu phản ứng bị ngừng lại sau một thời gian ngắn thì sản phẩm chính là sản phẩm được tạo thành nhanh hơn (sản phẩm khống chế động học – D). Tuy nhiên, phản ứng này xảy ra trong cân bằng và nếu thời gian đủ lâu, sản phẩm chính là sản phẩm bền về mặt nhiệt động học hơn – E.

241

5. Chuỗi chuyển hóa 1) Hoàn thành chuỗi chuyển hóa sau đây:

2) Hoàn thành chuỗi chuyển hóa sau đây:

3) Hoàn thành chuỗi chuyển hóa sau đây:

4) Hoàn thành chuỗi chuyển hóa sau đây:

242

Hướng dẫn 1)

2)

243

3)

4)

Từ sơ đồ trên có thể thấy rằng trong tổng hợp các hợp chất carbonyl thế ở vị trí α thì các enamine đóng vai trò là chất tổng hợp thay thế cho các anion enolate, và các phản ứng β-alkyl hóa, β-acyl hóa, và cộng hợp Michael (phản ứng Stork) diễn ra khi có sự tham gia của các enamine trong điều kiện trung hóa, điều này làm giảm tối đa các quá trình ngưng tụ phụ.

244

6. Xác định hợp chất tự nhiên Dựa vào các thông tin sau, hãy xác định cấu trúc hợp chất tự nhiên X, chất này tham gia vào việc điều hòa nhiều quá trình sinh lí liên quan đến huyết áp và các phản ứng dị ứng. -

a) b) c) d) e) f) g) h)

Hợp chất X có thể được xem là heptanoic acid có nhóm thế với khối lượng phân tử 354u và không chứa nguyên tử carbon bậc bốn. Hợp chất X không tạo thành phenylcarbazone. Phản ứng ozone phân của hợp chất X, được thực hiện trong môi trường có tính oxi hóa, tạo thành ba hợp chất (A, B, C). Đốt cháy 1.32 g hợp chất X, tương ứng với 10 milimol thu được 2.2 g CO2 và 0.72 g H2O. Hợp chất X không có tâm lập thể và khi được đun nóng sẽ tạo thành anhydride vòng D. Hợp chất B có thể được tạo thành ở dạng hỗn hợp racemic trong phản ứng của n-hexanal với HCN, và sau đó thủy phân sản phẩm tạo thành Z, quá trình được thực hiện trong môi trường acid. Cho hợp chất C phản ứng với CrO3, tạo thành hợp chất bên. Cho hợp chất C phản ứng với 2 đương lượng acetyl chloride thì thu được dẫn xuất F. Khử hợp chất C bởi lithium aluminium hydride thu được hợp chất G. Xác định cấu trúc của A. Xác định cấu trúc hợp chất D. Xác định cấu trúc các hợp chất Z và B. Xác định các cấu trúc có thể có của hợp chất C và chọn ra cấu trúc phù hợp chất với điều kiện đã cho trong đề bài. Vẽ một trong các đồng phân lập thể có thể có của hợp chất X. Chỉ rõ các tâm lập thể. Dựa vào số lượng các tâm lập thể, hãy xác định số xuyên lập thể phân (đồng phân dia) có thể có của hợp chất này. Vẽ các đồng phân lập thể có thể có khác của hợp chất X (bỏ qua các đồng phân quang học). Xác định cấu trúc hợp chất F và G.

245

Hướng dẫn a) Hợp chất A: b) Hợp chất D:

c) Hợp chất Z và B lần lượt là:

d) Các cấu trúc có thể có của C:

Dựa vào thông tin ở ý thứ 7 thì không thể xác định cấu trúc của hợp chất C bởi hợp chất này có 4 nhóm, mà một vài hoặc tất cả chúng có thể được tạo thành bởi sự oxi hóa hợp chất C bởi CrO3. Thông tin ở ý thứ 2 cho phép loại trừ các cấu trúc C2-C6. Vậy, cấu trúc duy nhất thỏa mãn đề bài là C1. e)

246

f) 25 = 32 xuyên lập thể phân. g) Đặt R= -CH2CH2COOH, và R’= -CH(OH)(CH2)4CH3

h) Cấu trúc F và G lần lượt là:

247

7. Ngưng tụ benzoin 2 mol benzaldehyde phản ứng được với 1 mol hợp chất A, sử dụng xúc tác ion cyanide. A tồn tại ở dạng cặp đối quang. Hợp chất A phản ứng với sodium borohydride tạo thành hợp chất B. B tồn tại ở dạng 3 đồng phân thể. a) Viết cơ chế phản ứng tạo thành A và phương trình phản ứng tạo thành B. Xác định các cấu trúc của A và B. b) Biểu diễn A, B ở dạng công thức tứ diện. Gắn các cấu hình R, S (theo quy tắc Cahn-Ingold-Prelog) với mỗi chất. Khi tiếp xúc với Al2O3, hợp chất B có phản ứng tách nước tạo thành C1 và C2. C1 và C2 tồn tại trong một cân bằng với C3. Trong phổ IR của C1 và C2 có các dải hấp thụ quanh vùng 1600 cm-1, và cả C1 và C2 đều làm mất màu nước bromine. Nếu xử lí B với sulfuric acid đặc để tách nước thì một phản ứng hoàn toàn khác xảy ra. Hợp chất D được tạo thành với một dải hấp thụ mạnh ở quanh vùng 1700 cm-1. Hợp chất D phản ứng với semicarbazide (H2N-NH-CO-NH2) tạo thành một hợp chất tương đối kém tan. c) Viết phương trình phản ứng tạo thành các hợp chất C1-C3 và D. Xác định cấu trúc của chúng và viết cơ chế phản ứng tạo thành D. Hợp chất D bị khử tạo thành hợp chất E. d) Viết cấu trúc của hợp chất E. Nếu xử lí E với acid mạnh thì phản ứng tách nước diễn ra tạo thành hợp chất F. Ozone phân F rồi xử lí tiếp với tác nhân oxi hóa tạo thành sản phẩm G (C13H10O). e) Xác định cấu trúc của F và G. f) Gọi tên G.

248

Hướng dẫn a)

b)

249

c)

D là một aldehyde, thể hiện hấp thụ hồng ngoại đặc trưng ở khoảng 1700 cm-1. Nó tạo thành một semicarbazone kém tan. d)

e-f)

250

8. Tổng hợp hữu cơ 1) Từ acetaldehyde, acetone, benzaldehyde và các tác nhân cần thiết khác, hãy điều chế 3-hydroxy-3-methyl-5-phenylpentanal – một chất có mùi hoa huệ thung lũng, được dùng làm nước hoa. 2) Đề xuất quy trình tổng hợp 5-hydroxypentanoic acid lactone từ cyclohexanone và các tác nhân cần thiết khác. 3) Từ isobutyraldehyde (2-methylpropanal), piperidine và methyl vinyl ketone, cùng các tác nhân cần thiết khác, hãy điều chế 4,4dimethylcyclohexen-2-one. 4) Điều chế hợp chất dưới đây từ bromobenzene, cyclohexanone và các tác nhân cần thiết khác:

5) Điều chế oct-6-yne-2-one từ 5-hydroxypentan-2-one, methylacetylene và các tác nhân cần thiết khác. 6) Từ diethyl malonate và ethylene oxide, cùng các tác nhân cần thiết khác, hãy điều chế ester vòng  -butyrolactone của 4-hydroxybutanoic acid. 7) Từ acetone, ethyl acetoacetate và các tác nhân cần thiết khác, hãy điều chế hợp chất A:

251

Hướng dẫn 1)

2)

3)

Phản ứng của enamine (hợp phần nhường, Michael donor) với methyl vinyl ketone (hợp phần nhận, Michael acceptor) là một ví dụ điển hình của phản ứng Michael. 4)

Phản ứng oxi hóa của các allyl và benzyl cũng có thể được tiến hành bởi tác động của MnO2 trong methylene chloride.

252

5)

6)

7)

253

9. Tổng hợp hữu cơ 1) Thực hiện chuyển hóa sau:

2) Đề xuất phương pháp tổng hợp 1-benzyl-2-methyl- và 1-benzyl-3methylcyclohexane, đi từ 4-methylcyclohexanone, benzyl alcohol, toluene và các tác nhân cần thiết khác. 3) Điều chế 3-phenylcyclohexanone và 2-phenylcyclohexadiene-1,3 từ cyclohexanone, bromobenzene cùng các tác nhân cần thiết khác. 4) Từ acetone và các tác nhân cần thiết, hãy điều chế 3,3-dimethyl-2oxobutanal. Điều gì sẽ xảy ra khi xử lí chất này với dung dịch potassium hydroxide đặc nóng? 5) Đề xuất sơ đồ tổng hợp dicarboxylic acid không no A từ cyclohexanol, triphenylphosphine, và ethyl ester của bromoacetic acid, cùng các tác nhân cần thiết khác.

6) Điều chế 1,3,3-trideutero-1-methylcyclopentane từ cyclopentanone, methyl iodide, cùng các tác nhân cần thiết khác.

254

Hướng dẫn 1)

2)

3)

255

4)

5)

6)

256

10. Cơ chế phản ứng Đề xuất cơ chế cho các phản ứng sau: a)

b)

c)

d)

257

Hướng dẫn a)

b)

c)

d)

258

11. Cơ chế phản ứng Đề xuất cơ chế cho các phản ứng sau: a)

b)

c)

d)

e)

259

Hướng dẫn a)

b)

c)

d)

260

e)

261

12. Cơ chế phản ứng Đề xuất cơ chế cho các phản ứng sau: a)

b)

c)

d)

e)

262

Hướng dẫn a)

b)

263

c)

d)

e)

264

13. Cơ chế phản ứng Đề xuất cơ chế cho các phản ứng sau: a)

b)

c)

d)

265

Hướng dẫn a)

b)

c)

266

d)

267

14. Cơ chế phản ứng Tiến hành phản ứng giữa 2,2-dibromohexanecarboxylic acid với 4 mol BuLi trong THF ở -78oC thu được chất A rất hoạt động. Sau đó người ta thêm từng giọt benzene đã được acid hóa (với sự có mặt của chất hút ẩm là silicagel) vào môi trường phàn ứng, tiếp theo cho từng giọt chất lỏng B vào và đun nóng thì người ta phân lập được chất cuối có công thức như sau. Đề nghị cơ chế của các phản ứng đã xảy ra.

Hướng dẫn

268